MTTC 103 Elementary (671Q)

Pataasin ang iyong marka sa homework at exams ngayon gamit ang Quizwiz!

questioning that focuses on factual questions and not open-ended questions.

didactic

The teacher wants to ensure the students gain adequate factual information from the informational text. Which of the following would be best

didactic questioning

Electrostatic attraction

Styrofoam cup rubbed against hair and put next to water

Question on Historical interpretation.

"George Washington did this well..."

rhetorical question

A question asked merely for effect with no answer expected.

Mark is studying a substance. Which property of the substance is chemical? A. density B. temperature C. melting point D. flammability

Answer: D Explanation: Chemical properties can be measured or observed only when matter undergoes a change to become an entirely different kind of matter. This includes reactivity, flammability, and the ability to rust

Concepts About Print (CAP)

Basic understand about ways to print

Nonlocomotor

Bending, twisting, stretching, rocking, swaying

Xylem and phloem in plants resembles what in mammals?

Blood vessels

There are many types of clouds. Out of the 4-main types of clouds, which clouds are mid level, fluffy, dense, mostly composed of water droplets? A. Stratus B. Cirrus C. Cumulus D. Nimbus

C. Cumulus

Directional Concepts

Children learn: that print is written and read from left to right on a page and from top to bottom

What helps a historian from plagiarizing?

Direct quotation marks and reference points.

Who is Dmitri Mendeleev? Explain his contribution to science.

Dmitri Mendeleev devised the periodic classification of the chemical elements, in which the elements were arranged in order of increasing atomic weight.

Which of the following best defines encoding?

Encoding refers to building words with sounds.

subtropics rates of evaporation

Evaporation of ocean water leaves behind the salts in the tropical regions.

Newton's Second Law

Force equals mass times acceleration

Which of the following was a major effect of the New Deal in the United States?

Increased federal government spending

What is the difference between invertebrates vs. vertebrates?

Invertebrates - animals without a backbone. Vertebrates - animals with a backbone.

Which activity would help allow children to be more creative?

Listening to music and finger painting how the music makes you feel.

Semantics

Meaning of words and sentences

Which two countries have had n issue with religious boundaries in the past?

Pakistan and India.

Instruction in phonics

Phonemic awareness in context of oral language tasks

What would base ten blocks be used for?

Place value

Which of the following strategies is the best to help students learn more appropriate oral language skills in group interactions?

Play recordings of positive group collaboration and have students analyze productive conversatio

Building with comprehension

Pre-reading titles and sidebars, looking at pics, predicting, asking questions, and making personal connections.

vocab development

Prior knowledge, defining words, using multiple examples, context clues, and breaking apart words.

Math in heredity

Probability

Consensus data

Researching and social structure and ethnic composition

Increase in oxygen levels in the atmosphere

Spread of photosynthetic life

In the 1800's why did Europeans( Irish) come to America?

The Irish Famine

Question about events that happened first

The Treaty of Paris- 1782 The Monroe Doctrine- 1823 The Missouri Compromise- 1830

which of the following best describes how the US democracy differs from democracy in Athens, Greece?

The US elects representatives as oppose to voting on laws themselves.

Spatial organization

The location of places, people, and events, and the connections among places and landscapes.

Encoding

The process of converting oral language into written language.

astronomy

The study of the moon, stars, and other objects in space

light waves

Transversed and can travel through a vacuums.

What can you do when a kindergartener is having difficulty with the alphabetic principle A is not sound A?

Use good communication and good comprehension

How would you measure a football field?

Using Square Kilometers.

Elkonin boxes

a series of boxes drawn on a sheet of paper, one box for each phoneme (sound) in a given word

scientific discipline

biology, neuroscience, psychology, and sociology.

Rotation of the Earth

causes day and night

Precommunicative Stage

children have not discovered that letters represent sound

kinetic energy

energy of an object in motion; find velocity with this

What is the Mode?

is the value that appears most frequently in a data set

sound waves

longitudinal travel through solid, liquid, and gas. They cannot travel through a vacuum

topographic maps

natural and human made surface features such as names of lakes, and towns, river courses, and roads

homogeneous

of the same kind

what is the study of biology and geology?

paleontology

Automaticity

refers to processing information at a competent level without requiring conscious effort.

There is a questions that shows a music piece and it asks what type of math goes with this.

relative magnitude of fractions

locomotion

the ability to move from place to place

Estuaries

the tidal mouth of a large river, where the tide meets the stream, enclosed coasts; Areas where fresh water rivers meet salt water areas.

reflectional symmetry

when one half is a mirror image of the other half

active listening...

"oh yeah, the same thing happened to me last week"

On Monday, Bobby sells 4 cakes for every 20 customers who come to his bakery. On Friday, he sells 12 cakes for every 60 customers. Assuming the same rate continues in the next days, how many cakes would he sell for 120 customers? A. 24 B. 18 C. 14 D. 1

A

Fable

A brief story that leads to a moral, often using animals as characters

complex sentence

A sentence with one independent clause and at least one dependent clause, (ex. When we buy his birthday cake, we have to make sure its lemon)

War of 1812

A war between France and Britain caused blockades that hurt American trade and caused the British to attack American ships and impress sailors on them. Britain became aggressive and war resulted/ The British captured Washington D.C. and burned the White House. In 1815 British ended the war with France and negotiated for peace with the United States under the Treaty of Ghent.

A teacher has the kids put an ice cube on top of saran wrap and place it in front of the window. This is modeling what? A. Experiment B. Observation C. Hypothesis

A. Experiment it doesn't mention them watching the process it just mentions setting up the experiment

Which of the following describes the most appropriate method for assessing a student's knowledge of the names and dates of major battles of the Civil War? a. a multiple-choice test b. a research report c. a debate d. a portfolio

A; a multiple-choice test would quickly assess student understanding of content knowledge.

Which of the following most contributed to the fall of the ancient Egyptians? a. a death of Cleopatra b. Alexander the Great conquering their land c. severe droughts D. Canaanite settlers entering Egypt

A; after Cleopatra's death, ancient Egypt became part of the Roman Empire.

Miss Randle is gathering information about her students at the beginning of the school year to determine their reading levels. She wants to find out what skills they are entering school with. Which assessment tool should she choose? a. diagnostic assessment b. formative assessment c. summative assessment d. aptitude test

A; diagnostic assessments pre-test students before a learning experience.

Which substance is a good insulator? a. glass b. water c. silver d. aluminum

A; glass is a good insulator.

Which planet does not have a moon? a. Mercury b. Earth c. Jupiter D. Saturn

A; only the first two planets, Mercury and Venus, lack moons.

Which skill must students master before learning the distributive property? a. multiplying whole numbers b. dividing integers c. multiplying decimals d. dividing fractions

A; the distributive property involves multiplying whole numbers.

A first-grade class cannot meet the unit's objective of writing a reflective essay on the book "Roll of Thunder, Hear My Cry." What is most likely the problem? a. the objective is developmentally inappropriate for these students. b. The teacher's instructional pacing was too fast, preventing mastery. c. The students are not engaged in the content; the teacher should select a different text. d. The teacher most likely did not sequence lessons within the unit effectively.

A; the novel and the essay are too difficult for first grade students.

The framers instituted a system of checks and balances because they were concerned about a. one branch of government gaining too much power. b. mob rule. c. the military taking over the government. d. the states overpowering the national government.

A; the purpose of checks and balances was to prevent tyranny in any branch of government.

Which of the following would be LEAST effective for teaching students about amphibians? a. looking up and copying definitions in the dictionary b. sorting animal cards based on characteristics c. dissecting a frog d. making a word map that represents the habitats of amphibians

A; there is no evidence supporting the effectiveness of copying definitions of terms to learn new vocabulary.

Which of the following are major physical regions in the continental United States? a. humid continental, humid subtropical, Mediterranean, marine, desert b. humid continental, humid subtropical, taiga, tundra, Mediterranean c. humid continental, Mediterranean, marine, taiga, tropical rainforest d. the Sun Belt, the Bible Belt, and the Coasts

A; these are all major physical regions in North America.

Newton's First Law

An object at rest stays at rest and an object in motion stays in motion with the same speed and in the same direction unless acted upon by an unbalanced force.

How long does it take for the moon to complete one full orbit around the earth? A. ~30 days B. ~60 days C. ~1 year D. ~2 years

Answer: A Explanation: The time period for the moon to complete one full orbit around the earth is ~30 days.

Which of the following sets of words is most closely associated with if/then text structure? a. since, result, consequently b. while, yet, similarly in addition, moreover, additionally d. such as, by, more

Answer: A Explanation: If/then structure is typically what happens if something happens. This is linked to the words since, result, and consequently.

Which of the following is the largest asteroid? A. Ceres B. Icarus C. Eros D. Vesta

Answer: A Explanation: Ceres is the largest object in the asteroid belt that lies between the orbits of Mars and Jupiter, slightly closer to Mars's orbit.

How long does it take Earth to spin around on its axis one time? A. 24 hours B. 7 days C. 31 days D. 365 days

Answer: A Explanation: It takes 24 hours for the Earth to spin around on its axis one time.

Which TWO of the following will result in reduced energy consumption? A. When replacing light bulbs, ensure that there are LEDs. B. Returning used light bulbs to manufacture for reuse. C. Shutting down computers at the end of the day. D. Leaving the engine of gasoline-powered car turned on when the car will be parked for more than 3 minutes.

Answer: A and C Explanation: Option A and C will support the cause of reducing energy consumption.

An elementary English teacher has read an information text regarding the Declaration of Independence to her students. The teacher wants to ensure the students have gain adequate factual information from the informational text. Which of the following would be the best to focus on? a. inquiry-based assessment b. didactic questioning c. at-home research project d. informal questionnaire assessment

Answer: B Explanation: Didactic questioning focuses on factual questions and not open-ended questions. Since the reading is related to social studies, which contains a lot of facts, using didactic questions is the best approach.

A teacher asks her kindergarten students to say words sound by sound while moving balls for each sound. This approach is showing what instructional method? a. intensive instruction b. explicit systematic instruction c. differentiated instruction d. shared reading

Answer: B Explanation: Explicit instruction is making the skill obvious to the student. Systematic instruction is following a sequence of development for language and reading acquisition.

Which of the following words are appropriate to include in a lesson on the different phonemes created by r-controlled vowels? a. angry b. bird c. real d. bridge

Answer: B Explanation: The /ir/ sound in "bird", similar to the same sound in "stir" and "third", is an r controlled sound.

Only eukaryotic cells have which of the following? A. DNA B. membrane-bound organelles C. organelle D. cytoplasm

Answer: B Explanation: Only eukaryotic cells have membrane-bound organelles.

The four planets closest to the sun are different from the next four planets in our solar system. The reason for this is because A. the four closest planets have more stars. B. the closest planets are more dense. C. the four closest planets have lesser diameters. D. the four closest planets take longer to complete five orbits.

Answer: B Explanation: The four planets closest to the sun are different from the next four planets in our solar system as the closer plants are denser.

The ________ has no liquid water, no atmosphere, no plate tectonics, and no global magnetic field. A. sun B. moon C. earth D. star

Answer: B Explanation: The moon has no liquid water, no atmosphere, no plate tectonics, and no global magnetic field.

A second grade class is learning about different fruits. The teacher asked the students to keep track of the number of apples, oranges, and pears they eat in a week for one month. The teacher wants students to create a visual to display the number of fruits eaten each week for one month. Which of the following will best display the information? a. flow chart b. concept map c. bar graph d. pi-graph

Answer: C Explanation: A bar graph can show how many fruits were eaten each week. The other options are not appropriate for displaying the information presented in the questions.

Which term best describes the prewriting process? a. structured b. rigid c. informal d. careless

Answer: C Explanation: Prewriting stage includes discussing notes and jotting down notes, which is best linked to the word informal.

monolog - shipment - happily Which of the following most accurately describes the words shown above? a. They contain prefixes. b. They are derived from German roots. c. They have multiple meanings. d. They contain suffixes.

Answer: D Explanation: Each of the words have a suffix. Monolog has "log." Shipment has "ment." Happily has "ly.,,

Art question about Julio Gonzalez, Little Dancer piece and how children should know what it is

Art question on test

Stacey is paid $20 per hair cut plus $3 of tips per customer. How many customers did she have if she made $207? A. 8 B. 9 C. 10 D. 11

B

The federal government and state governments in the United States exercise which of the following concurrent powers? a. determining voter qualifications b. establishing courts c. ratifying amendments to the U.S. Constitution d. conducting elections

B

The variable x is decreased by 50 and then multiplied by 3. The final result is 300. What is the value of x? A. 100 B. 150 C. 300 D. 400

B

Which of the following instructional strategies most directly supports content literacy? A. use of both expository and narrative texts in a thematic unit B. use of the K-W-L teaching model C. encouragement of comprehension monitoring during reading D. all of the above

B Although option A describes a good instructional strategy, it is not the best option, because it is too broad. It reflects a strategy that would promote literacy development in general, not just in the area of content literacy. Option C is also too general, because it describes a strategy that teachers should encourage during all sorts of reading, not just reading in content areas. Option D is incorrect, because option B is more directly supportive of content literacy than the other options. That is, option B is the best answer. The K-W-L teaching model encourages students to reflect on what they know and want to learn before reading an expository text and to consider what they learned after reading it. (See discussion of Competency 007 for further details.) Option B is the best answer in part because it is the only option that focuses specifically on comprehension of expository texts.

What is 5/8 as a percent? a. 16% b. 62.5% c. 1/6% d. 0.625%

B; 5/8 = 0.625, which is 62.5%.

Every morning, Mr. Kahn posts a morning message for students that contains errors in Standard English. Students read the message, identify the errors, and make suggestions for corrections. Mr Kahn is modeling a. paraphrasing b. editing c. decoding d. inferring

B; Mr. Kahn is modeling the editing process and reinforcing concepts related to the conventions of Standard English.

Which of the following are considered the elements of art? a. line, shape, color, and proportion b. line, shape, color, and form c. balance, unity, proportion, and texture d. line, texture, unity, and aesthetics

B; all of these are the elements of art.

A fourth-grade teacher places students into groups of four to six for reading activities. Each group works together to pick a book. They read each chapter independently, and then discuss the chapter as a group. The teacher assigns each member of the group a different role in the discussion. Which instructional method is the teacher using? a. direct instruction b. cooperative learning c. Jigsaw d. Socratic Method

B; group and student-led learning are the key markers of cooperative learning.

Which of the following is a feature of a hop? a. Alternate legs move in front of one another. b. Using the ball of one foot, a person pushes off from the ground on one leg. c. The body uses one foot to push off the ground and lands on the other. d. Both legs are bent; the toes push off the ground at the same time.

B; hopping also requires students to keep their heads forward and arms out. The leg that is not pushing off the ground is usually slightly bent.

Which of the following is an example of human geography? a. studying climate b. studying the spread of Christianity across the world c. studying the effect of land features on animals and plants d. studying continental movement over a period of time

B; human geography studies the relationship between humans the the physical world; studying how Christianity spread worldwide would need to address this relationship.

Kim and Chris are writing a book together. Kim writes twice as many pages as Chris. Altogether, there are 240 pages in the book. Which equation shows how many pages Chris writes? a. 2 + 2p = 240 b. p + 2p = 240 c. 2p - p = 240 d. p - 2p = 240

B; if p is the number of pages Chris writes, the equation shows that Kim writes 2p, or twice as many pages. If the number of pages Chris writes is added to the number of pages Kim writes, the total is 240 pages.

How did Martin Luther King and other activists achieve civil rights for black Americans? a. through violent uprisings b. using nonviolence c. with the help of the Soviet Union d. by lobbying Congress only

B; nonviolence was a defining characteristic of the mainstream Civil Rights Movement.

A teacher is designing a sequence of instruction for teaching letter recognition skills to emergent readers. Which of the following sets of letters would be most appropriate for a teacher to include in the first lessons? a. c, o, w, x, y, z b. I, L, T, H, E, F c. a, m, t, s, f, d d. A, a, B, b, C, c

C

During textbook reading, a sixth-grade teacher instructs students to carefully read the first and last sentence in each paragraph and to notice bold print and graphics. By engaging students in this activity, the teacher is applying which of the following strategies for promoting comprehension of expository texts? a. making inferences based on evidence presented in a text b. distinguishing fact from opinion in a text c. determining the importance of information presented in a text d. identifying structural elements in a text

C

If x = 9.322 and y is the number obtained by rounding x to a whole number, then what is x - y = A. 0.00322 B. 0.0322 C. 0.322 D. 3.22

C

Which of the following oral responses most clearly indicates that critical listening has occurred? a. "Are donations to your organization tax-deductible?" b. "How many people have donated money to your organization?" c. "How does your organization use private donations?" d. "I can't donate money to your organization, but I can donate my time."

C

Which of the following words would be most appropriate to include in phonics instruction designed to promote beginning readers' ability to decode regular CVCe words? a. done b. give c make d. were

C

Which of the following words would be most appropriate to teach as a sight word to fifth-grade students? a. teacher b. understand c. scientist d. explain

C

Which natural resource is a fossil fuel? a. water b. wind c. coal d. sun

C; coal is a nonrenewable resource that comes from the remains of plants that lived in swamps millions of years ago.

Which of the following is an essential element of descriptive writing? a. a plot arc b. reasoned judgment c. sensory words d. a graphic

C; sensory words are essential to descriptive writing because they are necessary for creating imagery.

In the fall, 425 students pass the math benchmark. In the spring, 680 students pass the same benchmark. What is the percent increase in passing scores from fall to spring? a. 40% b. 55% c. 60% d. 80%

C; to calculate the percent increase, find the difference between the ending number and the beginning number, and divide the difference by the beginning number. Then, multiply the decimal by 100 to change to a percentage.

You are packing your shirts into bags to move to another apartment. Each bag holds 7 shirts. If you are packing the 74th shirt, what bag are you filling? A. 8 B. 9 C. 10 D. 11

D

When asking children to tell a story using a textless book, which of the following should a kindergarten teacher expect? A. grammatical errors B. irrelevant details C. nonstandard English D. all of the above

D Option A is correct, because grammatical errors such as overregularization are normal among kindergarten students. Option B is correct, because limitations in narrative and pragmatic skills sometimes result in children speaking off-topic. Option C is also correct, because Nonstandard English is a common occurrence in the classroom. Hence, option D is the best answer.

Solve for x, if x = 6(3^0). a. 0 b. 18 c. 130 d. 6

D; any number with a zero exponent equals one.

A table is 150 centimeters long. How many millimeters long is the table? a. 1.5 mm b. 15 mm c. 150 mm d. 1500 mm

D; one centimeter = 10 millimeters and 150 x 10 = 1500.

What is the primary purpose of repeated reading? a. to build vocabulary b. to build concepts of print c. to build prediction skills d. to build fluency

D; repeated reading helps students gain a feel for how fluent reading feels and sounds.

The students in Mr. DeLuna's class investigate if there is a relationship between the weather and the number of bus riders. What data do the students need to collect? a. the mean number of bus riders during the first week of the month b. the number of rainy days this month c. the number of students who walk home from school when it is sunny d. the mean number of bus riders on rainy days this month

D; the students need to know the average number of bus riders on rainy days and compare that quantity to the average number of bus riders on days with fair weather.

A kindergarten teacher gives students picture cards of various activities carried out throughout the day; she then asks the students to sort the pictures into groups and explain their reasoning. The teacher is most likely trying to assess a. whether the students can determine which activities will take more and less time during the day b. whether the students can match digital time cards to each picture card c . whether the students can match each picture card to an analog time card d. whether the students can identify which activities are done during each time of the day (morning, afternoon, night)

D; the students will sort the picture cards into morning, afternoon, and night groups; then they will explain why each card belongs in its group.

Which of the following is a positive effect of urbanization? a. increased tenement housing b. sanitary conditions c. political machines d. increased employment opportunities

D; urbanization itself is the development of urban areas due to the arrival of job seekers; employment opportunities are themselves pull factors.

Balloon going up and air coming out

Every action, equal and opposite reaction( Newtons third law)

Why do Canadians live close to the U.S. border?

Freedom, supply of agriculture, climate, and water.

Question about what English errors exist.

French and Poodle both need to be capitalized.

What site should a gym teacher take an image off to avoid copyright?

Government website

MiPhy provides...

Health risk behaviors

The teacher wants the students to prepare a speech in 30 seconds. What does this help the students learn?

How to prepare a speech in concise terms.

What makes a region?

Human and Physical features

x>1

Image of a graph

control group

In an experiment, the group that is not exposed to the treatment; contrasts with the experimental group and serves as a comparison for evaluating the effect of the treatment.

Why is the ocean water in the subtropics more saltier?

In general, the sea is saltier in the subtropics, where evaporation is high due to warm air temperatures, steady trade winds, and very low humidity related to atmospheric circulation

In music, what are bar lines?

In music theory, a bar (or measure) is a single unit of time containing a specific number of beats played at a particular tempo.

Informal Assessment

Non-standardized tests such as criterion referenced test and teacher-prepared test. No rigid rules or procedures. (ex. running records, observations, or portfolios)

What is the longest river in Europe?

The Volga

Decoding

The process of converting written language into spoken sounds.

Why do you set goals in gym?

They are goals you can achieve (realistic).

Which of the following most accurately describes the composition of Earth's atmosphere? (A) Mostly nitrogen and oxygen (B) Mostly nitrogen and carbon dioxide (C) Mostly oxygen and carbon dioxide (D) Mostly oxygen and hydrogen

This question asks you to test your knowledge of the components of Earth's atmosphere. Dry air is approximately 78% nitrogen and 21% oxygen, with small amounts of other gases, such as approximately 0.039% carbon dioxide and 0.000055% hydrogen. The correct answer, therefore, is (A).

Historical Interpretation

When a certain historical event is described from different points of views.

Civic-mindedness

When a person puts the common good before their own interest.

Market Economy

When decisions regarding investment, production and distribution are based on market determined supply and demand.

When is the best time to be lobbying politician?

When you're trying to influence an election.

O-Zone layer

a layer in the stratosphere that contains a concentration of ozone sufficient to block most ultraviolet radiation from the sun

Transition Sentence

a sentence that leads the reader smoothly from a commentary sentence into quoted material in a paragraph

pluralistic society

a society made up of many different groups

the students try to match the baby animal (lower case) to the mama animal (upper case) for each letter of the alphabet. This is what?

alphabetic principle

Alphabetic Principle

an understanding that letters and letter patterns represent the sounds of spoken words.

Question about beats per measure in music

answer will be relative magnitude of fractions.

Rime

are "chunks" of words that always looks & sounds the same, a part of the word you can say (make, brake)

Diphthongs

are represented by two vowel letters where two sounds are heard.

Which of the following is not a diagraph?

fl

Didactic questioning

focuses on factual questions and NOT open-ended questions.

Between 1880 and 1920 it was a time of?

industrialization and urbanization.

Which term best describes the prewriting process?

informal

Phonological awareness

is the acknowledgement of sounds and words; for example a childs realization that some words rhyme.

Which of the following concepts involves understanding how words are formed, and their relationship to other words in the same language?

morphology

Students spell words using...

phonemes

context clues

refers to the text surrounding an unfamiliar word.

what activity will allow students to engage more regarding fact and opinion statements

repetition

Heterogenesis

reproduction in which successive generations are different

Sound segmentation

requires the student to progress from isolating one sound to isolating all the sounds within the word.

•Sound substitution

requires the student to subtract, add , or substitute sounds in presented words.

Rhyme

sound the same, but may look different at the end (make, break)

Which of the following must be accomplished by students before learning vowel diphthongs?

spell regular words

Formal Assessment

standardized written or performance test of knowledge, aptitude, values, etc.

Compounds

2 or more elements chemically combined, and composed of atoms

analysis

A detailed examination of the elements or structure of something.

Elements

A molecule composed of one kind of atom; cannot be broken into simpler units by chemical reactions.

If x=5, what is the value of the algebraic expression 2x - x? a. 5 b. 10 c. 15 d. 20

A; 2(5) - 5 = 5.

Which of the following are examples of figurative language? a. metaphor and personification b. rimes and onsets c. roots and affixes d. facts and opinions

A; metaphor and personification are types of figurative language.

Ms. DiCristafaro is considering using peer assessment to help students edit their writing assignments. Which of the following steps should she take first? a. establish a safe learning environment b. pair students by ability level c. teach students active listening skills d. remind students of the importance of grades

A; students must feel safe and trust their peers to be able to participate in peer assessment.

Mr. Patrick would like his students to self-assess their history projects before turning them in. Prior to beginning the project, which of the following should take place? a. set learning goals b. locate resources c. critically evaluate their work d. articulate learning

A; students should write SMART goals to drive their learning experience.

Which of the following would be the most appropriate resource for a lesson on the aftermath of Germany after World War I? a. a description of the Treaty of Versailles b. a photograph of Adolf Hitler c. a chart detailing the total deaths of soldiers from different countries d. a video of the first nuclear explosion

A; the Treaty of Versailles had huge implications for the future of Germany.

Which equation demonstrates the associative property of addition? a. 2 + (1 + 5) = (2 + 1) + 5 b. 2(1 x 5) = (2 x 1)5 c. 1 x 3 = 3 x 1 d. 2(7 + 4) = 2 x 7 + 2 x 4

A; when using the associative property, the answer will remain the same in an addition problem regardless of where the parentheses are placed.

Below is passage from the story Pecos Bill (1966). What Bill planned to do was leap from his horse and grab the cyclone by the neck. But as he came near and saw how high the top of the whirling tower was, he knew he would have to do something better than that. Just as he ... came close enough to the cyclone to feel its hot breath, a knife of lightning streaked down into the ground. It struck there, quivering, just long enough for Bill to reach out and grab it. As the lightning bolt whipped back up into the sky, Bill held on. When he was as high as the top of the cyclone, he jumped and landed astraddle its black, spinning shoulders. By then, everyone in Texas, New Mexico, Arizona, and Oklahoma was watching. They saw Bill grab hold of that cyclone's shoulders and haul them back. They saw him wrap his legs around the cyclone's belly and squeeze so hard the cyclone started to pant. Then Bill got out his lasso and slung it around the cyclone's neck. He pulled it tighter and tighter until the cyclone started to choke, spitting out rocks and dust. All the rain that was mixed up in it started to fall. The above best represents which of the following literary genres? a. tall tales b. epics c. myths d. fairy tales

Answer: A Explanation: A tall tale is a story with unbelievable and highly improbable elements, related as if it were true and factual.

Which of the following is example of heterogenous mixture? A. milk B. glue C. blood D. pizza

Answer: D Explanation: Homogeneous mixtures refer to compositions that are the same throughout. Heterogeneous mixtures are two or more substances that are distinct from one another. Pizza is heterogeneous mixture.

A type of clam known as a leaf shell has not been discovered alive in over 100 years. This clam is best described as which of the following? A. threatened B. historical C. protected D. extinct

Answer: D Explanation: Since the clam has not been found for over 100 years, the clam is extinct. Extinction is the termination of an organism or of a group of organisms, usually a species.

Editorial writing

Article that presents an opinion of an issue, develops that idea in the body, and offers a solid and cons use conclusion that summarizes the narrators opinion

When a magnet is heated to room temperature what happens?

At around 80 °C, a magnet will lose its magnetism and it will become demagnetized permanently if exposed to this temperature for a period, or if heated above their Curie temperature. Heat the magnet even more, and it will melt, and eventually vaporize.

If x + 2z =10, x/y = 2, and y =2, then what is the value of z? A. 2 B. 3 C. 4 D. 5

B

There are many types of clouds. Out of the 4-main types of clouds, which clouds are high level, wispy, curly, composed of ice crystals? A. Stratus B. Cirrus C. Cumulus D. Nimbus

B. Cirrus

Which of the following could be used as an informal assessment for determining a student's understanding of solid shapes? a.. a worksheet where students color each shape in a different color b. a teacher's observations and notes about each student during geometry work stations c. a multiple-choice test d. a group project about solid shapes

B; teacher observations would be a valuable informal assessment in a classroom where students discuss shapes and their attributes during hands-on activities, and use reasoning skills when explaining their creations.

If 9x + 9 = 18(y + 6), then 3x - 6y = A. 27 B. 30 C. 33 D. 36

C

Plant height in a particular species of plant is determined by a single gene. Plants with two copies of the dominant form of the gene (HH) grow to be about 3 feet tall. Plants with two copies of the recessive form of the gene (hh) grow to be only 1 foot tall. Which of the following will be the most likely distribution of plant heights in 100 offspring of a cross between a 3-foot-tall HH plant and a 1-foot-tall hh plant? a. All of the offspring will be 2 feet tall. b. Half of the offspring will be 3 feet tall and half will be 1 foot tall. c. All of the offspring will be 3 feet tall. d. Three-quarters of the offspring will be 3 feet tall and one-quarter will be 1 foot tall.

C

The product of two numbers is 72 and one of the numbers is . Find the sum of the two numbers. A. 18 B. 20 C. 22 D. 24

C

Which of the following themes is frequently explored in the works of Julia Alvarez, Mildred Taylor, and Laurence Yep? a. the excitement of exploring other galaxies b. the yearning to live in harmony with nature c. the determination to overcome social injustice d. the challenge of confronting personal setbacks

C

Mark's 1st-grade teacher notices that although he has good handwriting, Mark sometimes complains that his hand hurts when he writes. What should the teacher do first? A. Send Mark to the school nurse. B. Call Mark's parents to get their feedback. C. Observe Mark's pencil grip closely. D. Ignore Mark's complaints unless they persist

C Option D is clearly incorrect, because a child's physical complaints should not be ignored. Options A and B are not the best initial actions on the teacher's part. Although it is important to refer students to the nurse if a physical problem is suspected and to keep lines of communication with parents open, the first thing to do in this particular case is to simply check whether Mark is holding the pencil too tightly, "hooking" his wrist, or doing something else as he writes that could be easily corrected

A 2nd grader would benefit most from structural analysis of which of the following difficult words? A. jacket B. sneaker C. underwear D. tuxedo

C Structural analysis allows children to identify morphemic constituents of words. Option C, unlike the other options, contains a word that is composed of two simpler words. By breaking the word down into "under" and "wear" the child may be able to read the entire word more easily. The other words in the list cannot be broken down into simpler morphemic units. Hence, option C is the correct answer.

Which best states the purpose of the rewriting stage of the writing process? a. presenting outcomes b. organizing ideas c. correcting errors d. recording thoughts

C; correcting errors is the purpose of the rewriting stage of the writing process.

Which organism is a primary consumer? a. mushroom b. corn c. cow d. lion

C; cows eat plants but do not eat other animals; therefore cows are primary consumers.

Which of the following is a significant contribution of ancient China? a. Mahjong b. toothbrushes c. gunpowder d. the animal zodiac

C; gunpowder is one of the most important inventions of ancient China.

Which is the primary reason elementary students learn how to paraphrase and cite information from outside sources? a. to learn the names of authors b. to identify secondary sources c. to avoid plagiarism d. to condense writing

C; learning the importance of avoiding plagiarism is a critical component of writing instruction for school and career success.

Which of the following is true of quantitative measures of text complexity? a. They are task considerations determined by professional judgment. b. They are analytical measurements determined by knowledge demands. c. They are statistical measurements determined by computer algorithms. d. They are leveling measurements determined by text structure.

C; quantitative measures are objective and based on statistics.

Book Orientation Concepts

Children learn:how to hold a bookhow to turn the pagesthat print, not the illustrations, carry the meaning

A teacher is planning a student concert at which students will perform a published song from a contemporary musical. Given the ethical considerations associated with student performances of published material, which of the following actions by the teacher would be most appropriate? a. purchasing one copy of the sheet music, photocopying it for students, and destroying the copies after the concert b. including an acknowledgment of the song's composer and publisher in the concert program c. downloading the song from the Internet, burning CDs for students, and having students learn the song by ear d. sending a written request for permission to the song's publisher well in advance of the concert

D

As part of an informal reading assessment, a teacher asks a first-grade student to read a grade-level passage aloud. As the student reads, the teacher marks the student's errors on a copy of the passage, indicating errors of substitution, insertion, omission, and reversal. The teacher also notes any repetitions or self-corrections. This assessment is likely to provide the most useful information about the student's skills in which of the following areas of reading development? a. reading comprehension b. vocabulary knowledge c. phonemic awareness d. word recognition

D

What term is used when the moon moves between the earth and the sun? a. aurora b. lunar eclipse c. black hole d. solar eclipse

D; when the moon moves between the earth and sun, a solar eclipse occurs, blocking sunlight from the planet.

Science and technology are different in the fact that technology

Deals with application of science to solve problems

Question about shoe company and using machinery to create goods for a lower price.

Law of Diminishing returns- He can make a bigger profit by limiting labor

How to find the area of a square

LxW

morpheme

The smallest meaningful units of language.

All of the following are likely to affect the height of an ocean wave as it nears the shore EXCEPT (A) wind speed (B) wind duration (C) topography of the seafloor (D) water temperature

This question asks you to analyze the effect of various factors on the height of an ocean wave. The wind provides energy to form waves. Waves are high when wind speeds are high, when the wind blows for a long period of time, and when the wind blows in the same direction over a long distance. When waves approach shore and enter shallow water, wavelength decreases and wave height increases, and breakers may form. Water temperature has little effect on wave height. The correct answer, therefore, is (D).

If a jet travels due east from New York City to a final destination 1/3 of the way around Earth, the jet would pass through approximately how many time zones? (A) 3 (B) 8 (C) 24 (D) 36

This question asks you to apply your knowledge of Earth's rotation and time zones. It takes 24 hours for Earth to rotate once on its axis, and there are thus 24 time zones. One-third of 24 is 8. The correct answer, therefore, is (B).

"The earth's vegetation is part of a web of life in which there are intimate and essential relations between plants and the earth, between plants and other plants, between plants and animals." This passage is from Silent Spring (1962), by Rachel Carson, a book about the destructive effects of DDT. Carson's "web of life" refers to which of the following scientific concepts? (A) Food chain (B) Evolution (C) Ecosystem (D) Extinction

This question asks you to interpret a passage about science and connect it to an important scientific concept. Rachel Carson was concerned that the chemical pesticides, for example, DDT, introduced by humans into the environment would have a devastating effect on the interactions of organisms with each other and the environment. This relationship is an example of "ecosystem," an ecological community together with its environment, functioning as a unit. (C), therefore, is the correct answer

Of the following planets, which has been most thoroughly investigated for evidence of water and life by instrumentation sent to its surface from Earth? (A) Mercury (B) Mars (C) Jupiter (D) Saturn

This question asks you to use your knowledge of exploration of the solar system. Spacecraft have been sent to get information about other planets, but the probes usually pass by the planets without landing. The exploration of Mars has been more intensive, with several spacecraft landing on Mars's surface. For example, rovers have been sent to Mars, and the instruments on them have been used to gather and send back a great deal of data. The correct answer, therefore, is (B).

anecdotal record

a report of a child's actions that concentrates on a specific behavior or area of development

~partner students and have them listen to one another read~ask students to record themselves while reading~students to read the same text...

boosts fluency

The boy is 4 ft tall with a shadow of 6. The tree is 30 ft tall, how tall is the shadow?

boy/shadow=tree/x

elementary teacher wants to promote student's ability to make healthy choices by

building opportunities for students to make decisions into everyday class activities

A plant being able to open and close during day and night is a result of

diffusion or dehydration

In music, what is melody?

the aesthetic product of a given succession of pitches in musical time, implying rhythmically ordered movement from pitch to pitch.

Dolch List

the most well known basic sight word list

sound blending

the student must pull individual sounds together to make a word.

unreliable sources

untrustworthy materials from a person or institution that does not have the educational background, expertise, or evidence of legitimate sources to support a claim (ex. wikepedia, twitter, facebook)

Syllables

• The ability to blend, segment, and delete syllables.

Concept of spoken word

• The ability to distinguish words in a sentence.

Phonemes

• The ability to recognize initial and final sounds in words

Vowel Digraphs

• Two vowel letters that make one sound.

Expository writing is writing that seeks to explain, illuminate or 'expose'. This type of writing can include essays, newspaper and magazine articles, instruction manuals, textbooks, encyclopedia articles and other forms of writing, so long as they seek to explain. Expository writing differs from other forms of writing, such as fiction and poetry.

Fiction Genre, gives background info

Which of the following is done first in theatre? A. practice lines B. assign roles

B. assign roles The first rehearsal usually involves a "table read" of the script where each person simply says their lines in character. You must have a role first to read your practice lines.

Which of the following best describes what vaccines have done? A. keep humans age B. allowed humans to become immune to diseases C. prevent diseases

B. allowed humans to become immune to diseases

The library buys 2,932 books and an employee donates 302 books. Six students check out 5 books each and lost them. What is total amount of books the library has? A. 2548 B. 2600 C. 2675 D. 3204

D

Explain Fair Use Act in Classroom.

Fair Use is an important copyright concept for educators who use copyrighted works in their teaching. The Fair Use doctrine permits limited use of copyrighted material without acquiring permission from the rights holders.

What was Michigan's role in WWII?

Making War equipment

Diffraction

Occurs when an object causes a wave to change direction and bend around it

Weathering

The breaking down of rocks and other materials on the Earth's surface.

The tilting of the earth creates what?

seasons

core values of US political culture

* Legal equality * Political equality * Freedom of religion * Freedom of speech

What is the function of the U.S. President?

- Make sure all rules are carried out and governments run effectively. - veto power over laws passed by legislative. -commander in chief for military; he can deploy them.

How to promote language aquisition in the classroom.

- words walls, read out loud, anchor charts, diverse classroom library, and daily convos.

If, 4x = (12x + 3) x 3, then x equals A. 3/32 B. - 3/32 C. 9/32 D. - 9/32

D

who developed the theory of earths rotation?

Nicholas Copernicus

What is the greatest effect on family when a member is addicted to drugs?

Loss of interest in usual activities.

Math question ( cube with squares placed in different areas).

The answer is Spatial!

What is a market system?

A market system is the network of buyers, sellers and other actors that come together to trade in a given product or service.

steps in the writing process

1. Prewriting- think and decide( brainstorm) 2. Drafting- Write(read, write, repeat) 3. Revising- Make it better( read, change, take out or add, replace things) 4. Edit 5. Evaluating- Make it correct( complete sentences, correct spieling, change words that are not used right) 6. Publishing

What equation would work best for this problem: There are 15 times as many students as teachers.

15(S)= T

The dominate gene is B and has brown fur, the recessive gene is b and has white fur. What is the percentage that they child will have have white?

25%

A fourth-grade teacher is planning a unit in which students will use a computer application to create their own tessellations. The teacher would like to develop a series of related activities and assessments to help develop students' mathematical reasoning skills related to the use of the application. Which of the following topics would be most appropriate for the additional materials to address? a. reflections, rotations, and translations b. area of composite figures c. similarity and proportional reasoning d. graphs of simple equations

A

A kindergarten teacher wants to promote proper dental hygiene among kindergarten students. Which of the following strategies would be most effective for the teacher to use in addressing this goal? a. providing the opportunity and the supplies needed for students to brush their teeth after lunch b. sending dental hygiene pamphlets home with students and following up with a reminder to read them c. discussing with students the importance of brushing one's teeth after every meal d. suggesting to students that they should encourage their families to visit a dentist at least once a year

A

A sixth-grade teacher would like to help students conduct research on the nutritional components of their school lunch program. Which of the following resources would be most appropriate for the teacher and students to consult first? a the Web site for the Michigan Department of Education b. dietary guidelines on http://www.health.gov c. an article about the Healthy, Hunger-Free Kids Act of 2010 d. the USDA's ChooseMyPlate home page

A

A third-grade class has been studying the weather, including measuring the temperature, humidity, wind direction, and amount of rainfall every day. Which of the following strategies would be most effective for assessing the knowledge and skills of the students at the end of this weather unit? a. having each student use their collected weather data to explain the past weeks' weather in a written summary b. giving the students a written test in which they need to explain illustrations of various weather phenomena and weather data c. giving the students an assignment to make their own weather measurement tools, and use them to collect weather data at home d. having the students verbally explain which weather factors are most important to measure every day, and how best to measure them

A

Find the largest integer if the sum of three consecutive even integers is equal to 258. A. 87 B. 88 C. 89 D. 90

A

If 4x + 6y = 20 and x + y = 12, then what does 3x + 5y = A. 8 B. 10 C. 12 D. 14

A

The highest tides usually occur during a full or new moon primarily because at that time the: a. sun, moon, and earth form a straight line. b. earth/moon system is closest to the sun. c. axis of the earth is pointed directly at the moon. d. moon is closest to the earth's equator.

A

Which of the following is an example of a way in which a living organism chemically changes matter? a. A plant uses energy from the sun to produce sugars. b. A fungus releases spores from fruiting bodies into the air. c. A fish obtains oxygen from the water through its gills. d. A chipmunk removes the shells from nuts before eating them.

A

Which of the following can be expected in a preschool classroom? A. Children struggle with the distinction between the way "b" and "d" are written. B. When children are shown pictures of a cat, a bat, and a hat, they find it difficult to point to the "cat." C. Children who know the names of the 26 letters also know how to write them. D. All children acquire graphophonemic knowledge at about the same pace.

A Option B is incorrect, because accurate performance relies on auditory discrimination skills that all preschoolers should have. Children with normal hearing will have no difficulty pointing to the "cat." A child who is unable to do so may have a hearing deficit, or some other problem that should be evaluated. Option C is incorrect, because it is unlikely that by preschool age most children know how to write all 26 letters. Likewise, Option D is incorrect, because individual differences in the acquisition of graphophonemic knowledge can be expected. Option A is correct. It is normal for preschoolers to experience some difficulty distinguishing between the way highly similarlooking letters are printed.

Which of the following activities is best for promoting the development of writing among 1st graders? A. Each child is encouraged to make a Christmas letter for a family member, describing something special that the child has experienced. The letter contains a mix of artwork and writing that the child creates. B. Each child keeps a notebook in which he or she writes each letter of the alphabet ten times. The notebook is assembled over a period of weeks and upon completion, children can see how their handwriting has progressed. C. Children are taught how to air-write each letter of the alphabet. Then, the class plays a game in which teacher sings the alphabet song very slowly while the children trace each letter in the air. D. Each week, the teacher focuses on the writing of a different letter. Children are encouraged to write, paint, and draw the letter using different media.

A Options B, C, and D may contribute to improvement in writing, but they focus on handwriting in isolation from a meaningful use of language. Option A is the best answer, because the creation of letters to relatives would allow children to practice writing about topics that interest them, and to incorporate the writing into meaningful communication

Arbirtration

A form of alternative dispute resolution in which a dispute is submitted to a third party or a panel of experts outside the judicial trial system

Totalitarianism

A form of government in which the ruler is an absolute dictator (not restricted by a constitution or laws or opposition etc.) The government attempts to maintain 'total' control over society, including all aspects of the public and private lives of its citizens. There are several characteristics that are common to totalitarian regimes, including: 1. Rule by a single party 2. Total control of the military 3. Total control over means of communication (such as newspapers, propaganda, etc...) 4. Police control with the use of terror as a control tactic 5. Control of the economy

Interdependence

A relationship between countries in which they rely on one another for resources, goods, or services

simple sentence

A sentence consisting of one independent clause and no dependent clause (I bought my friends some candy.)

There are many types of clouds. Out of the 4-main types of clouds, which clouds are low level, layers like a blanket, composed of water droplets? A. Stratus B. Cirrus C. Cumulus D. Nimbus

A. Stratus

Which of the following is an example of hand-eye coordination? A. Throwing a bean bad at a target B. juggling scarves

A. Throwing a bean bad at a target Juggling scarves does not involve vision. Any task that requires the coordination of vision and hand movements involves hand-eye coordination. Examples of hand-eye coordination include grasping objects, catching and throwing a ball, playing an instrument while reading music, reading and writing, or playing a video game.

Kinetic energy will increase and potential energy will decrease when? A. book falling to the ground B. air filling a balloon

A. book falling to the ground As the book is falling, its potential energy is converted to kinetic energy. Thus, PE is decreasing and KE is increasing. Air filling a balloon is PE increasing until the air is released

A first-grade teacher is looking to teach about space and wants to do a play. Which of the following is best to undertake? A. pretend to be aliens for the day B. memorize lines C. direct/conduct a play

A. pretend to be aliens for the day engage kids into wanting to get into character for a play. It is a good intro to the unit and gives the teacher time to organize how they want to go about setting up roles and such

Which number is equal to (5^2 + 1)^2 + 3^3? a. 703 b. 694 c. 30 d. 53

A; 703

Which medical technology can be used to detect cancerous tumors? a. MRI b. defibrillator c. EKG d. patient monitor

A; MRI stands for Magnetic Resonance Imaging and is used to observe organs and internal structures.

Which property is demonstrated by the equivalent expressions? 7(3 + 55) = 7 x 3 + 7 x 55 a. distributive b. associative c. commutative d. multiplicative

A; The equivalent expressions demonstrate the distributive property, which says that multiplication distributes over addition.

Which of the following would NOT be a good lesson to teach the principles of democracy in the United States? a. having all students vote publicly for class officers b. engaging students in a class discussion on evaluating the powers, duties, and limitations of state government offices c. taking students through a mock process that mimics how laws are passed in Congress d. presenting the system of checks and balances

A; US citizens vote anonymously.

A teacher notices that a group of students who have just finished reading the same novel are having difficulty identifying how the story transitions from one stage to another. Which of the following is the best graphic organizer to help students understand the story's development? a. a plot pyramid b. a Venn diagram c. a main idea/key details chart d. a sequencing chart

A; a plot pyramid helps students identify and visualize how an author moves a story forward in clear stages with specific characteristics; exposition, rising action, climax, falling action, and resolution.

Which of the following is an example of a formative assessment? a. a student portfolio b. a final exam c. a final research project d. a standardized test

A; a student portfolio is an example of a formative assessment that provides a perspective on a range of student work over time.

When would students need to understand how to find the least common multiple? a. when adding fractions with unlike denominators b. when adding fractions with unlike numerators c. when multiplying fractions with unlike denominators d. when multiplying fractions with unlike numerators

A; equivalent fractions with common denominators must be used to add fractions. The LCM helps the student find the equivalent fraction.

Which is the best writing style for a research paper? a. expository b. descriptive c. narrative d. persuasive

A; expository writing is most appropriate for the formal and objective presentation of information required in a research paper.

Which of the following assignments best assesses students' use of secondary and/or primary sources?

Writing an essay on the American Revolution impact on American society.

Ms. Noyes teaches a lesson on order of operations. What information do the students need to know before the lesson begins? a. how to simplify exponents b. how to plot a function on a graph c. how to write an equation in logarithmic form d. how to expand binomials

A; in order to solve operations expressions, students need an understanding of addition, subtraction, multiplication, division, and use of exponents.

Which of the following terms best describes a situation in which the federal government prevents the only Internet service company in a local community from raising its prices? a. economic regulation b. social regulation c. opportunity cost d. scarcity

A; in this instance of economic regulation, the government is controlling prices by preventing a monopoly.

Which of the following would be a good activity to teach students about the three states of matter? a. making root beer floats b. making homemade butter c. making ice cream in a bag d. blowing up a balloon

A; making root beer floats demonstrates the three states of matter: solid, liquid, and gas.

Which of the following can be classified as persuasive writing? a. an advertisement for a new product b. a research paper on the effects of climate on ecosystems c. a poem about the ocean on a foggy day d. a short story with a suspenseful plot

A; persuasive writing aims to influence the reader to agree with what is stated and to act accordingly.

Which of the following is an example of prosody? a. using appropriate vocal cues when reading aloud b. decoding words correctly when reading aloud c. reading at an appropriate speed when reading aloud d. reading smoothly and steadily when reading aloud

A; prosody is a reader's ability to use appropriate vocal expressions when reading aloud.

Asking citizens to be civil minded and independent and making the people as a whole sovereign is a characteristic of a. republicanism. b. government. c. economics. d. communism.

A; republicanism also stresses natural rights as central values.

Mr. Armendt, the English teacher, provides students with detailed descriptins of criteria that define each level of writing performance. Students use the descriptions to learn what they can do to be better writers and then the student's level of performance is graded according to the criteria. Which assessment tool is Mr. Armendt using? a. rubric b. selected response c. checklist d. anecdotal notes

A; rubrics are a fixed scale that measures performance with detailed descriptions of criteria that define each level of performance.

Which of the following is considered an unreliable research source? a. a self-published report b. an edited book of essays c. a newspaper article d. a university study

A; self-published materials are considered unreliable sources because expertise has not been established.

Which of the following is the primary purpose for playing sight word games with a group of students? a. to develop word recognition skills b. to develop oral language skills c. to develop social skills d. to develop phonics skills

A; sight word games are tools for developing word recognition skills in order to increase reading fluency.

A first grade teacher provides students a part/part/whole mat and cubes. The teacher asks the students to find a way to make the number 7 using the cubes and mat. The teacher is most likely helping the students to understand a. the different number pairs that will equal the number 7 b. how to identify the number of items in a set c. how to show conservation of numbers d. how to compare numbers

A; students are using the cubes to find different ways to make the number 7. The different ways can be listed on chart paper as number pairs that equal 7.

Matthew, a fourth grade teacher, gives his students ten minutes of computer time whenever they turn an assignment in on time. Matthew is using what kind of motivation? a. extrinsic motivation b. intrinsic motivation c. situated motivation d. achievement motivation

A; the computer time is an external motivator.

Which part of the word "dream" is a phoneme? a. ea b. dr c. eam d. re

A; the diphthong "ea" produces one small unit of sound, /e/, that cannot be reduced into a smaller unit.

A teacher has just provided students a general overview of a research project due at the end of the semester. Which of the following is most appropriate as a first task? a. to select a topic and locate some general information on the topic b. to develop an outline listing the main idea and supporting details c. to choose reliable primary and secondary resources d. to formulate an appropriate research question on a topic

A; the first step in any research project is to select a topic of interest and acquire background knowledge about it. Once general information on a topic is understood, a relevant question can be formulated.

A teacher asks, "What word am I trying to say, /p/ /i/ /n/?" and instructs students to say the word. Which strategy is the teacher using to build phoneme awareness? a. phoneme blending b. phoneme deletion c. phoneme segmentation d. phoneme substitution

A; the strategy of phoneme blending requires students to combine phonemes to make a word.

The students in Mrs. Mathis' class want to know the probability of pulling a yellow ball out of a jar that contains fifty balls. What information do the students need to know to find the answer? a. How many yellow balls are in the jar? b. How many different colors of balls are in the jar? c. How many purple balls are in the jar? d. How much do the balls cost?

A; the students need to know how many of the fifty balls in the jar are yellow.

The students in Mr. Burd's class want to know if attendance affects math test scores. What information do the students need to know? a. What is the mean attendance rate? b. What did Daryl make on the math test? c. How many times has Kerry been absent? d. Who made the lowest grade on the math test?

A; the students need to know the average attendance rate (and the average math score) for each attendance group.

How best could a teacher model inferences during reading? a. by thinking aloud b. by providing a graphic c. by drawing a plot pyramid d. by conducting a word investigation

A; thinking aloud is when a teacher verbalizes thoughts and insights to model how to think logically and critically when applying a reading comprehension skill.

The Saffir-Simpson _______ Wind Scale is a 1 to 5 rating based on a sustained wind speed. A. Tsunamis B. Hurricane C. Tornado D. Earthquake

Answer: B Explanation: Hurricane winds are measured using the Saffir-Simpson Hurricane Wind Scale.

Which of the following is an event that successfully ended the Cold War? a. the August coup b. the fall of the Berlin Wall c. the Cuban missile crisis d. the success of the Sputnik program

A; this 1991 event in which members of the Soviet government attempted to take control from Mikhail Gorbachev contributed to the dissolution of the USSR, effectively ending the Cold War.

Which of the following could be used as a culminating activity for a unit on weather to allow the students to use what they have learned? a. Assign students group projects on different types of storms. Students should provide an example of the storm and describe the characteristics and effects of the storm. b. Give an essay test on the types of clouds. Students list all of the types of clouds and describe their characteristics. c. Give students a multiple-choice test on all of the elements of meteorology. d. Assign students an end-of-chapter test in their science workbooks.

A; this activity would allow students to pair what they have learned with critical thinking and reasoning skills to complete their projects. Students can demonstrate their deep understanding of the material.

Which of the following quotes is a demonstration of active listening? a. "If I understand correctly, you think the classroom needs a pet." b. "I think you meant to say that you wish you had a pet at home." c. "You didn't talk about the kind of pet our classroom should have." d. "There's no way the principal will let us have a classroom pet."

A; this quote demonstrates active listening because the listener is paraphrasing the message to clarify understanding.

The teacher wants a more pluralistic society in her classroom library that involves more literate quality.

Add books with characteristics and events that are familiar to the child. The child will understand them.

Which of the following best defines the difference between decoding and encoding? a. Decoding refers to the process of reading — translating words into sounds and ideas. Encoding refers to building words with sounds. b. Encoding refers to the process of reading — translating words into sounds and ideas. Decoding refers to building words with sounds. c. Decoding refers to the process of writing — translating words into letters and ideas. Encoding refers to building words with sounds. d. Decoding refers to the process of reading — translating words into sounds and ideas. Encoding refers to building words with letters.

Answer: A Explanation: Decoding refers to the process of reading — translating words into sounds and ideas. Encoding refers to building words with sounds.

A teacher notices that a group of students cannot capture the big picture when reading an article related to the American Revolution. Which of the following is the best graphic organizer for helping students understand the big picture of information text? a. fact pyramid b. story map c. KWL chart d. semantic-features analysis matrix

Answer: A Explanation: Fact pyramids provide teachers with a structured way of analyzing information in textbooks to guide students' focus toward big ideas. Fact pyramid graphically categorize text information into three levels: (1) essential facts ;( 2) short-term facts; and (3) supportive detail.

Fourth-grade students are preparing to read a short story about two students' visit to Boston art gallery. Which of the following activities is the best way to introduce the text to promote reading comprehension? a. completing a KWL chart b. writing about their own experience visiting art gallery c. indentifying Boston on a map d. gaining basic facts about Boston

Answer: A Explanation: KWL chart allows the student to document what they know before reading and after reading, which allows students to build knowledge and support reading comprehension.

Which of the following concepts involves understanding how words are formed, and their relationship to other words in the same language? a. morphology b. graphophonic analysis c. phonemic awareness d. syntax

Answer: A Explanation: Morphology is the study of the internal construction of words. In particular, morphology is how they are formed, and their relationship to other words in the same language.

Jack is having some difficulty in reading as indicated by his score on an oral reading fluency assessment. When he reads, he sometimes inserts words that are not in the text. This shows a weakness in which area of fluency? a. prosody b. rate c. accuracy d. automaticity

Answer: A Explanation: Prosody refers to the expressiveness with which a student reads. The student can improve in prosody.

Which of the following sentences contains a relative clause? a. The band that came to the city last month was very good at playing music. b. After reviewing my homework, I notice I missed few problems. c. I hated my job for years, and I wanted to quit. d. I like to stay home, and I like to watch television.

Answer: A Explanation: Relative clauses are clauses starting with the relative pronouns who, that, which, whose, where, when.

A teacher is conducting a reading comprehension lesson in which the she is teaching subject/topic, main idea, and details. Which of the following option is best example to use with the students? A. Subject/Topic: high blood pressure Main Idea: Sodium can cause high blood pressure Detail: A lot of sodium results in water hold up that can increase pressure in blood vessel. B. Main Idea: high blood pressure Subject/Topic: Sodium can cause high blood pressure Detail: A lot of sodium results in water hold up that can increase pressure in blood vessel. C. Subject/Topic: high blood pressure Main Idea: A lot of sodium results in water hold up that can increase pressure in blood vessel. Detail: Sodium can cause high blood pressure D. Subject/Topic: Sodium can cause high blood pressure high blood pressure Main Idea: high blood pressure Detail: A lot of sodium results in water hold up that can increase pressure in blood vessel.

Answer: A Explanation: The main topic is related to high blood pressure while the main idea is concerning sodium causing high blood pressure. The detail in Option A is appropriate for the topic and main idea.

A first-grade teaching is spending a portion of the afternoon having students write and draw in personal journals. The students are free to write and draw about any topic they desire. This approach will mainly benefit students by promoting their ability to a. think creatively. b. think deeply. c. get personal. d. improve grammar.

Answer: A Explanation: The students are free to select what they want to draw or write, so the students have the freedom to think more creatively.

Which of the following correctly orders the layers of soil from the top to the bottom? A. humus, topsoil, elevation, subsoil, regolith, bedrock B. humus, topsoil, elevation, regolith, bedrock, subsoil C. regolith, humus, topsoil, elevation, subsoil, bedrock D. bedrock, humus, topsoil, elevation, subsoil, regolith

Answer: A Explanation: The following are layers of soil from the top to the bottom: humus, topsoil, elevation, subsoil, regolith, and bedrock.

Which of the following best shows the life cycle of strawberry bushes growing naturally in a forest? A. seed -- growth -- flower -- fruit B. seed -- flower -- growth -- fruit C. fruit -- flower -- seed -- growth D. fruit -- seed -- flower -- growth

Answer: A Explanation: The life of strawberry bushes growing naturally in a forest will start with a seed. Then, the growing process will start. Flowers will grow, and then fruit will grow.

What happens to red light when shined on a green leaf? A. it is absorbed B. it is reflected C. it is distributed D. it turned green

Answer: A Explanation: When red light is given to a leaf, it will absorb the red light and therefore appears as black.

Which of the following clouds never block out the sun completely, but rather produce a variety of optical effects? A. cirrostratus B. cumulus C. cirrus D. cirrocumulus

Answer: A Explanation: Cirrostratus clouds never block out the sun completely, but rather produce a variety of optical effects.

____________ is an egg cell and a sperm cell joined together to form a new cell called a zygote. A. Fertilization B. Gametes C. Sexual Reproduction D. Asexual Reproduction

Answer: A Explanation: Fertilization is the action of fertilizing an egg, female animal, or plant, involving the fusion of male and female gametes to form a zygote

Which of the following planets has very little atmosphere? A. Mars B. Venus C. Jupiter D. Neptune

Answer: A Explanation: Mars is a red planet with very little atmosphere.

Which of the following is a major advantage of asexual reproduction over sexual reproduction? A. Enables organisms to reproduce without a mate; no wasted time or energy. B. Offspring are identical, so there is no genetic variation. C. Traits can develop to resist harsh environments that allow for an organism to survive better. D. If the parent has the mutation in their DNA, then the offspring will have it too.

Answer: A Explanation: Of the choices, Option A is the major advantage of asexual reproduction over sexual reproduction. Option B and D are disadvantages of asexual reproduction. Option C is advantage of sexual reproduction.

Which of the following hormones is most directly involved in maintaining pregnancy, preparing the body for conception, and regulating the monthly menstrual cycle? A. progesterone B. testosterone C. estrogen D. estriol

Answer: A Explanation: Progesterone is a hormone released by the corpus luteum in the ovary. Progesterone has an important role in the menstrual cycle and in maintaining the early stages of pregnancy.

Jenny is conducting an investigation about the basic needs of a plant. She gives four plants different amount of soil and the same amount of sunlight and water. Jenny measures the growth of the plants. What is the manipulated variable in this investigation? A. soil B. water C. light D. growth

Answer: A Explanation: Sunlight and water are remained constant while soil is changing, so the manipulated variable is soil.

Paula is conducting an investigation about the basic needs of a plant. She gives four plants different amount of soil and the same amount of sunlight and water. Paula measures the growth of the plants. What is the manipulated variable in this investigation? A. soil B. water C. light D. growth

Answer: A Explanation: Sunlight and water are remained constant while soil is changing, so the manipulated variable is soil.

Which of the following is in order from smallest to largest? A. solar system, galaxy, universe B. solar system, universe, galaxy C. galaxy, solar system, universe D. universe, solar system, galaxy

Answer: A Explanation: The smallest to largest is the solar system, then the galaxy, and then the universe.

____________ is the process by which DNA is copied to mRNA, which carries the information needed for protein synthesis. A. Transcription B. Mutation C. Replication D. Translation

Answer: A Explanation: Transcription is the process by which DNA is copied to mRNA, which carries the information needed for protein synthesis.

What form of energy makes a compass needle point north? A. magnetic B. light C. current D. electrical

Answer: A Explanation: A magnetic compass points north because it aligns itself with the magnetic field produced inside Earth.

Which of the following would most likely cause an object to change its speed? A. friction B. gravity C. mass D. pushing an object downhill

Answer: A Explanation: Friction will cause an object to change speed.

An elementary science teacher places three small cubes on a table in front of a science class. Each cube is the same size and painted the same color. Each cube is made of a different material: copper, iron, and wood. Which of the following tools would best help the students identify the cube made of iron? A. bar magnet B. balance beam C. hot plate D. magnifying lens

Answer: A Explanation: Iron is a metal, so it has magnetic properties. Option A can help students determine which cube is iron.

Which TWO best indicates the differences between active transport and passive transport? A. Active transport is the movement of materials through a membrane that requires energy to move molecules in the opposite direction of the way molecules move naturally. B. Passive transport is the movement of materials through a membrane that does not require energy because the molecules are moving through holes in the cell membrane until the same number of molecules are on both sides of the cell membrane. C. Passive transport is the movement of materials through a membrane that requires energy to move molecules in the opposite direction of the way molecules move naturally. D. Active transport is the movement of materials through a membrane that does not require energy because the molecules are moving through holes in the cell membrane until the same number of molecules are on both sides of the cell membrane.

Answer: A and B Explanation: Active transport is the of ions or molecules across a cell membrane into a region of higher concentration, assisted by enzymes and requiring energy. Passive transport is the movement of materials through a membrane where energy is not require.

Which THREE of the following safety practices should be implemented by an elementary education science teacher? A. Always wear splash-proof safety goggles when working with, or observing, students who are using chemicals, hot liquids, or flying objects. B. Wear a laboratory apron made of cotton, if there is a chance of soiling or damaging clothing. C. Remind students to keep science materials away from their mouths, noses, or eyes when conducting or cleaning after an activity. D. Throw all paper towels or other clean-up materials or chemical wastes in normal trash bins.

Answer: A, B, and C Explanation: All paper towels and clean up chemical wastes should be in a separate container for chemical wastes. Do not put chemical wastes in the normal trash containers.

Which of the following words best describes the ending of a comedy? a. shocking b. happy c. sad d. conflict

Answer: B Explanation: A comedy is to assume or delight the audience, and the action in a comedy usually ends happily.

Since the weather is bad, I'll stay inside for the remainder of the night. The above sentence is which of the following? a. simple sentence b. complex sentence c. compound sentence d. compound-complex sentence

Answer: B Explanation: A complex sentence is a sentence with an independent clause joined by an dependent/subordinate clause.

Which of the following sentences contains an error in punctuation? a. The girls went out to the store to buy fruits: apples, orange, pairs, and grapes. b. The baseball game was a very interesting event as many people were there c. I went to the shop, but I later decided to go to the movies. d. I really think it is time to make a change in my life; however, making a change is difficult.

Answer: B Explanation: A period is needed at the end of the sentence.

Which of the following is not a diagraph? a. st b. fl c. ch d. wh

Answer: B Explanation: Diagraphs are "voiceless" combinations of two consonants. Option B is a consonant blend.

A fifth-grade elementary teacher gives her students an article titled "Exploring Native American Culture." The teacher wants to assign a pre-reading activity that will improve students' understanding and learning while they read the article. Which of the following is the best approach to take? a. review technical terms b. discuss headings and illustrations c. research on Native Americans on Internet d. discuss their own culture

Answer: B Explanation: Discussing headings and illustrations will allow the students to get a sense of what they will read, which will support them in understanding and learning while they read the article.

Megan, an elementary education teacher, has a group of students that have trouble interacting with peers during group discussions. Which of the following strategies is the best to help students learn more appropriate oral language skills in group interactions? a. Show students a presentation on communication skills to use in various environments. b. Play recordings of positive group collaboration and have students analyze productive conversations. c. While working in groups, give immediate feedback to correct oral language skills. d. Work individually with each student struggling with oral language skills.

Answer: B Explanation: Getting students exposed to appropriate group interaction is a good strategies to employ. Moreover, the students are going to analyze the conversation, which they can recall during future group discussions.

Which of the following is most accurate regarding fiction and nonfiction text? a. Fiction can be both narrative and informational. b. Nonfiction can be both narrative and informational. c. Fiction can be informational. d. Nonfiction can be informational only.

Answer: B Explanation: Narrative tells a story. The main character or person in the text faces a problem and tries to resolve the problem. There is a beginning, middle, and end. Informational text is written to describe factual information in an understandable format. Nonfiction can be both narrative and informational while fiction can only be narrative.

Which of the following assignments best assesses students' use of secondary and/or primary sources? a. Reading an article from a scientific journal regarding experiment related to birds. b. Writing an essay on the American Revolution impact on American society. c. Writing a personal narrative explaining how school uniform should be banned. d. Writing a journal entry on the field trip visit to NASA Space Center.

Answer: B Explanation: Option B is the only option that directly requires the use of secondary and/or primary sources.

Below is excerpt from "The Raven" (1845) by Edgar Allen Poe. "Once upon a midnight dreary, while I pondered, weak and weary, Over many a quaint and curious volume of forgotten lore, While I nodded, nearly napping, suddenly there came a tapping, As of some one gently rapping, rapping at my chamber door. Tis some visitor," I muttered, "tapping at my chamber door - Only this, and nothing more." The repetition of similar word sounds creates a mood of a. cherry joy. b. increasing tension. c. growing relaxation. d. jubilation.

Answer: B Explanation: Poe continuously repeats the sounds (e.g. "-apping") to create a sense of urgency and increasing tension.

An elementary reading teaching is looking to get her students involved in self-grading their own reading skills. Which of the following is the best activity to accomplish this goal? a. choral reading b. taped reading c. echo reading d. buddy reading

Answer: B Explanation: Taped reading is when the teacher tapes the children reading individually. This can be done with the microphone on an interactive whiteboard or a tape recorder. The children can listen back to what they have read. They self-assess their own reading.

Which of the following word identification strategies typically is the most advance? a. using phonic knowledge b. analyzing word structure c. applying context clues d. identifying diphase vowel

Answer: B Explanation: The most advance word identification strategies is analyzing word structure. Analyzing involves using critical thinking skills.

~writes uppercase and lowercase letters ~writes own name ~uses invented spellings to express meaning ~uses invented spellings to write teacher-dictated words The above skills are achieved at what grade level? a. Pre-K b. Kindergarten c. First d. Second

Answer: B Explanation: The skills outlined indicate achievement at the kindergarten level.

A fifth-grade teacher models how to select unknown words and write an explanation, reread areas where they struggle to understand, write questions based on their passages, and infer what will happen in the next passage. These are examples of comprehension strategies that include: a. clarifying, summarizing, questioning, and predicting b. clarifying, monitoring, questioning, and predicting c. defining, monitoring, previewing, and predicting d. clarifying, monitoring, questioning, and previewing

Answer: B Explanation: The students are asked to write explanation, which is linked to clarifying. The students reread areas they struggle with, which require monitoring weak area development. Writing questions is linked to questioning strategy. Inferring what will happen next is linked to predicting.

Which of the following must be accomplished by students before learning vowel diphthongs? a. pronouns and nouns b. spell regular words c. spell words that rhyme d. understand syllabication

Answer: B Explanation: Vowel diphthongs are characteristic of irregularly spelled words, so students must learn to spell regular words before going into vowel diphthongs.

Which structure of a dolphin has the same function as the gills of a fish? A. fin B. lung C. heart D. mouth

Answer: B Explanation: Instead of having lungs to breathe, a fish has gills. Lungs and gills have the same purpose; they both supply the body with oxygen. The lung of the dolphin has the same function as the gills of a fish.

How can humans best help endangered species avoid extinction? A. They can provide shelter for the animals. B. They can protect the animals' habitat. C. They can provide more food to the animals. D. They can put the animals in a zoo.

Answer: B Explanation: Protecting the animals' habitat is the best to help endangered species avoid extinction.

Time to time, trees are purposely removed from forests. What is the purpose of removing these trees? A. to build homes B. to reduce the risk of fires C. to prevent erosion in the forest D. to improve the soil quality of the forest

Answer: B Explanation: The purpose of removing tress in forests is to reduce the risk of fires.

Which of the following instruments is best used to measure the volume of a small irregularly shaped solid? A. triple beam balance B. graduated cylinder C. thermometer D. ruler

Answer: B Explanation: With the object being irregular and looking to find the volume, the best method is using graduated cylinder.

_________ are underground rocks that store groundwater, which include sand, gravel, or limestone. A. Sedimentary rocks B. Aquifers C. Lava D. Sandstone

Answer: B Explanation: Aquifers are underground rocks that store groundwater, which include sand, gravel, or limestone.

Lizard and snake are both reptiles. Which of the following questions about an animal's physical characteristics is best to use when determining whether an animal is a reptile? A. How tall is it? B. Does it have a backbone? C. What color is the skin? D. How rigid is the skin?

Answer: B Explanation: Characteristics of a reptile is having a backbone, ability to produce eggs, scales, and cold-blooded

Which of the following is NOT one of the four major parts of the Earth that work together as a complex system? A. atmosphere B. ecosystem C. biosphere D. geosphere

Answer: B Explanation: Four major parts of the Earth that work together as a complex system are geosphere, hydrosphere, atmosphere, and biosphere. An ecosystem is a community of living organisms interacting with one another; this also includes nonliving components of the environment.

Which of the following measures how much water vapour is present in the air as a gas? A. toxicity B. humidity C. precipitation D. concentration

Answer: B Explanation: Humidity is a measure of the how much water vapour is present in the air as a gas.

The Saffir-Simpson _______ Wind Scale is a 1 to 5 rating based on a sustained wind speed. A. Tsunamis B. Hurricane C. Tornado D. Earthquake

Answer: B Explanation: Hurricane winds are measured using the Saffir-Simpson Hurricane Wind Scale

Which of the following tectonic processes forces rocks up from beneath the earth's surface? A. melting B. uplifting C. deposition D. crystallization

Answer: B Explanation: Uplift is a tectonic process that forces rocks onto the earth's surface.

Which of the following cells have a concave shape that helps them to carry oxygen molecules? A. guard cells B. red blood cells C. nerve cells D. amoeba cells

Answer: B Explanation: Red blood cells have a concave shape that helps them to carry oxygen molecules. Nerve cells have extensions that connect to other nerve cells to make an information network. Amoeba cells have an irregular shape that allows them to surround food to digest. Guard cells are shaped to help open and close plant stomata.

While working in a laboratory, Layth dropped a glass test tube filled with an unknown liquid. What should Layth do first? A. put on gloves B. inform the teacher C. sweep up the broken glass D. ask a friend to watch area to get cleaning supplies

Answer: B Explanation: The first action Layth needs to take is inform the teacher. The teacher will know how to handle the situation.

Because the chicken was too cold, I warmed it in the microwave. The above is example of which type of sentence? a. run-on b. compound c. complex d. compound-complex

Answer: C Explanation: A complex sentence combines a dependent clause with an independent clause. When the dependent clause is placed before the independent clause, the two clauses are divided by a comma; otherwise, no punctuation is needed.

__________ is an inclination or tendency towards an idea. a. Cohesion b. Claim c. Bias d. Push

Answer: C Explanation: Cohesion is the flow of sentences, paragraph, or sections of text to show connection among ideas. Claim an arguable statement. Bias is an inclination or tendency towards an idea.

Which of the following is not a consonant blend? a. bl b. br c. ar d. spl

Answer: C Explanation: Consonant blends (also called consonant clusters) are groups of two or three consonants in words that makes a distinct consonant sound. Option C is not a consonant blend.

An elementary education teacher is using an informational text regarding polar bears. When selecting vocabulary from the informational text, which of the following approaches is most beneficial? a. selecting long words b. selecting technical terms c. asking students to select words d. using experience to select words

Answer: C Explanation: Having students select words to further study will be targeted learning toward their needs.

A sixth grade teacher wants to support her students in becoming more aware of their literacy development and better monitor development in reading. To best support this goal, the teacher should have the students a. compile assignments and assessments in a portfolio. b. complete a daily assessment record at the end of the day. c. set detail goals regarding areas of improvement needed. d. develop rubrics to peer assesses one another.

Answer: C Explanation: Having students set goals for improvement areas supports the idea of getting them involved in monitoring skills related to reading.

Which of the following sentence would be most appropriate for an introduction to a persuasive essay? a. Mr. Martin has been involved in all the research studies, and he is mean. b. Doing good research with Mr. Martin looks good on your resume. c. Mr. Martin may not be the nicest man, but he has two traits that make him a good researcher. d. Mr. Martin has a vast knowledge in research from his previous employment.

Answer: C Explanation: Option C provides a clear and concise sentence that gives an overview of what will be discussed in the writing piece. The sentence clearly sets the stage about how Mr. Martin is nice and has traits that make him a good researcher.

Which of the following strategies is the most beneficial for students during the publishing stage of the writing process? a. reviewing writing for spelling error b. checking formatting c. typing work on a computer d. ensuring table of content

Answer: C Explanation: Publishing is the final stage in the writing process, so typing the work on a computer is a great way to publish the final draft.

An elementary teacher is conducting a lesson on fact and opinion statements. Immediately after finishing the lesson, the teacher has the students complete a worksheet on identifying fact and opinion statements. All students performed above 90% on the worksheet. Few weeks later, the teacher conducted a unit test, and all the students scored below 70% in the section assessing fact and opinion statements. Which of the instructional strategies would have been beneficial to ensure and maintain learning? a. paired activity b. word wall c. repetition d. reciprocal teaching

Answer: C Explanation: Repetition of the activity will have allowed the student to engage more regarding fact and opinion statements, which would increase the chances of maintaining knowledge.

An elementary teacher has her students read the story "The Mitten." Then, she gives her students a worksheet with images of different animals, and she instructs her students to label the animals below in the order they were introduced in the story. Which of the following reading strategies is being most targeted in this activity? a. recalling b. retelling c. sequencing d. summarizing

Answer: C Explanation: Sequencing is a reading strategy that requires students to sequence the events in a story, or retell the story in chronological order.

While reading a book aloud, one of the students believes the teacher is reading the pictures instead of the words in the book. Which of the following is the student lacking in understanding? a. letter movement b. phonological awareness c. concepts of print d. alphabetic principle

Answer: C Explanation: Since the student thinks the teacher is reading the pictures as oppose to the words, the student lacks understanding in concept of print. Concepts of print refers to the awareness of how print works.

A sixth-grade student, Jimmy, who is a struggling reader will engage in an activity in which older students read aloud predictable or rhyming books to first grade students. This activity will most likely promote Jimmy's reading development in which of the following ways? a. develop Jimmy's foundational reading skills b. develop Jimmy's phonemic awareness c. improve Jimmy's reading fluency d. engage Jimmy in different subject topics

Answer: C Explanation: The key in the question is "predictable or rhyming" books, which contain multiple repetition of words and phrases. This allows the student to develop automaticity, accuracy, and prosody (appropriate expression), which are key reading fluency skills.

A fifth grade teacher shows her students a poster that advertises orange juice. The advertisement includes multiple short statements regarding orange juice. The teacher asks students to detect any faulty reasoning. Which of the following levels of reading comprehension is mainly being used in this activity? a. literal b. inferential c. evaluative d. appreciative

Answer: C Explanation: The students evaluate statements of orange juice to their own experience and knowledge, so the skill used in the activity is evaluative.

Which of the following represents the amount of mass contained per unit volume? A. mass B. weight C. density D. volume

Answer: C Explanation: Density is the amount of mass contained per unit volume.

Claire carried her hoverboard up a hill and then rode the hoverboard down the hill. When Claire reached the bottom of the hill, she came to a stop. When did Claire have the most potential energy? A. while carrying the hoverboard up the hill B. while riding the hoverboard down the hill C. while standing on the hoverboard at the top of the hill D. while standing on the hoverboard at the bottom of the hill

Answer: C Explanation: Potential energy formula is mass times gravity times height. The most potential energy is at the highest point, which is standing on the hoverboard at the top of the hill.

In which of the following body systems are the pituitary, thyroid, and parathyroids found? A. skeletal system B. nervous system C. endocrine system D. muscular system

Answer: C Explanation: Pituitary, thyroid, and parathyroids are found in the endocrine system.

Phytoplankton -->Zooplankton --> Herring -->Salmon In the above food chain, the zooplankton is ____________. A. tertiary consumer B. primary producer C. primary consumer D. secondary consumer

Answer: C Explanation: Primary consumers are herbivores, which mean they eat plants. Secondary consumers may be carnivores (meat eaters) or omnivores (eat plants and animals). Tertiary consumers are carnivores that eat other carnivores

Which of the following is an example of a prokaryotic cell? A. virus B. liver cell C. bacterium D. none of the above

Answer: C Explanation: Prokaryotes are unicellular organisms that lack organelles or other internal membrane-bound structures. Bacterium is a prokaryotic cell.

I. located in the nucleus, or center, of the atom II. have single positive charge III. have some mass Of the above, which of the following are shared by protons and neutrons? A. I and II B. II and III C. I and III D. I, II, and III

Answer: C Explanation: Protons and neutrons are located in the nucleus, or center, of the atom. The proton has a single positive (+) charge. The neutron has a zero (0), or neutral, charge. The proton and neutron have approximately the same mass

I. located in the nucleus, or center, of the atom II. have single positive charge III. have some mass Of the above, which of the following are shared by protons and neutrons? A. I and II B. II and III C. I and III D. I, II, and III

Answer: C Explanation: Protons and neutrons are located in the nucleus, or center, of the atom. The proton has a single positive (+) charge. The neutron has a zero (0), or neutral, charge. The proton and neutron have approximately the same mass.

Composition: like a ball of rock and dirty snow made of rock, iron and frozen water and gases Size: up to about 5 km Orbit: long, elliptical orbits that take them from beyond the orbit of Uranus to around the sun Which of the following align with the information above? A. Mars B. Jupiter C. Comet D. Asteroid

Answer: C Explanation: The description in column A best describes a comet.

What is the reason that causes the oceans to bulge toward the moon? A. earth's gravity B. sun's gravity C. moon's gravity D. tides

Answer: C Explanation: The pull of the moon's gravity causes the oceans to bulge toward the moon.

During the cooling process, igneous rocks form from which of the following? A. magna B. lava C. both magna and lava D. neither magna nor lava

Answer: C Explanation: When lava cools and crystallizes, lava becomes igneous rock. Igneous rocks form as magma cools underground.

The pH of a substance can be determined by which of the following? A. thermometer. B. conductivity tester. C. litmus indicator D. pH test

Answer: C Explanation: Litmus indicator solution turns red in acidic solutions and blue in alkaline solutions. It turns purple in neutral solutions.

Which of the following is NOT an open syllable word? a. no b. my c. fly d. hat

Answer: D Explanation: An open syllable has a vowel at the end of the syllable. A closed syllable is a vowel followed by a consonant.

Which of the following is not written in the first person? a. autobiography b. memoir c. personal essay d. biography

Answer: D Explanation: Biography is written in the first person; it is typically written by someone else.

Kate loves her cats, and the cats love her too because she feeds them. The above sentence is which of the following? a. simple sentence b. complex sentence c. compound sentence d. compound-complex sentence

Answer: D Explanation: Compound-complex sentence contains at least two independent clauses (like a compound sentence) and at least one dependent clause (like a complex sentence).

I. partner students and have them listen to one another read II. ask students to record themselves while reading III. direct students to read the same text more than once An elementary education teacher is looking to boost fluency. Of the above, which of the following is/are ways to improve fluency? a. I and II b. I and III c. II and III d. I, II, and III

Answer: D Explanation: Fluency is defined as the ability to read with speed, accuracy, and proper expression. All options are ways to improve fluency.

More than 70% of a first grade class scored at high risk on the oral reading fluency assessment. Which instructional practice would be most effective for improving the students' oral reading fluency? a. daily independent reading in the afternoon b. peer reading on familiar text c. vocabulary development on unfamiliar words d. constantly reading text with corrective feedback

Answer: D Explanation: Having students read text and providing feedback will support students in oral reading fluency. Option D targets reading in addition to ensuring improvement with constant feedback.

The following excerpt is from "The Elements of San Joaquin" (1977), a poem by Gary Soto. At dusk the first stars appear. Not one eager finger points toward them. A little later the stars spread with the night And an orange moon rises To lead them, like a shepherd, toward dawn. Which poetic device is used in this excerpt? a. idiom b. metaphor c. alliteration d. simile

Answer: D Explanation: In the last two lines of the excerpt, a simile is

The following is conversation between a teacher and her students: Teacher: Please read this word. - Teacher will show a flashcard with the word "shop." Students: shop Teacher: Correct, the word is shop. Hector, how many letters in shop? James: S-h-o-p: four letters. Teacher: And how many sounds in shop, Blair? Blair: /sh/ /o/ /p/: There are two sounds. Teacher: How many sounds are in the word stop, Clair? Clair: There are three sounds. Teacher: How many sounds are in the word spin, Kate? Kate: There are none. Based on the conversation, which student has the least competency in blending sound? a. James b. Blair c. Clair d. Kate

Answer: D Explanation: Kate indicates that there are no sounds in the word "spin." This is clear indication that her skills related to blending sound is the lowest.

Knowledge of the relationship between letters and sounds is _ a. phonemic awareness b. fluency c. vocabulary d. phonics

Answer: D Explanation: Knowledge of the relationship between letters and sounds is phonics.

Understanding social and emotional aspect is linked mostly to which levels of reading comprehension? a. lexical comprehension b. literal comprehension c. interpretive comprehension d. affective comprehension

Answer: D Explanation: Lexical comprehension is understanding key vocabulary in text. Literal comprehension is answering who, what, where, and when questions. Interpretive comprehension is answering what if, why, and how questions. Affection comprehension is understanding social and emotional aspect.

Some people love cold winters near the fireplace with hot coffee. Others love the hot summer with snow cones and ice cream cones. Write an essay that names your favorite season and give reasons why it is best. The above prompt is associated with which type of writing? a. narrative b. descriptive c. expository d. persuasive

Answer: D Explanation: Persuasive writing attempts to convince the reader that the point of view recommended by the writer is valid. The prompt best represents persuasive writing.

Which of the following words is the best example to use when demonstrating structural analysis for vocabulary development? a. bike b. house c. maintain d. bicyclist

Answer: D Explanation: Structural analysis involves using prefixes, suffixes, and root words to determine the meaning of an unfamiliar word. The word "bicyclist" is the only word that has a prefix, suffix, and root word, so it would be the most useful in teaching structural analysis.

Read the sentence below, which contains a spelling error; then answer the question that follows. As time passes, dietitians get better at mapping healthier eating plans that effects our health. Which of the following words is misspelled in this sentence? a. dietitians b. mapping c. healthier d. effects

Answer: D Explanation: The correct spelling is "affects."

Which of the following is not one of the five essential components of reading identified by the National Reading Panel? a. phonics b. fluency c. vocabulary d. spelling

Answer: D Explanation: The five essential components of reading are phonemic awareness, phonics, fluency, vocabulary, and comprehension.

A teacher is going to print cards (one set for each student) and has them cut up the cards. Once the cards are all cut up, the students try to match the baby animal (lower case) to the mama animal (upper case) for each letter of the alphabet. The teacher is focusing on which of the following? a. concepts of print b. decoding c. thinking aloud d. alphabetic principle

Answer: D Explanation: The information gained from this activity is helpful in knowing where a student is with understanding alphabetic principle, which is the understanding that letters represent sounds which form words.

Below is a sample work from a sixth grade student: There are many sales going on during the Black Friday Holiday Sales. There are stores that provide many discounts on electronics and clothing. There can be stores that even offer free stuff during the holiday season. There can be so much fun during the holiday period. Which is not an area the student needs improvement in? a. word choice b. capitalization c. sentence structure d. spelling

Answer: D Explanation: The student needs improvement in capitalization due to him capitalization "holiday sales." Each of the sentences start with "there," so the student needs to work on word choices. All sentences used are basic, so sentence structure improvement is needed. No spelling error is shown in the student's work.

The genotype for a pea plant that is homozygous recessive for both height and pea color would be? A. tt B. YY C. ttYy D. ttyy

Answer: D Explanation: Consider the dominant alleles for height and pea color in a pea plant to be T and Y, respectively. In the same way, the recessive alleles for height and pea color are t and y, respectively. The homozygous genotype is characterized for having pairs of the same form of allele. For example: YY and yy are both homozygous, since the pair is formed by the same type of allele. Moreover, the genotype for the pea plant in question that is homozygous recessive for both height and pea color is ttyy.

A fifth-grade student who is having difficulty reading the word instinctively in a science text would be best advised to apply which of the following skills? a. word recognition b. structural analysis c. syllabication d. phonic analysis

B

A geographer would most likely use a remote sensing technology for which of the following purposes? a. to develop soil conservation plans b. to study climate variability c. to monitor the spread of a disease d. to create equal-area maps

B

A third-grade teacher plans to assess students' active listening skills by observing small-group discussions and using a checklist to record observations. Which of the following criteria would be most appropriate to include on the checklist? a. speaking at least 25 percent of the time b. rephrasing a speaker's statement before responding c. nodding and smiling during the discussion d. waiting to speak until other students take their turn

B

An individual wants to purchase an air conditioner and can choose between two different options. Both options have an original price of $200. The first option offers 30% off the original price and charges $20 for shipping. The second option offers 15% off the original price and shipping is free. What is the difference in savings between the two options? a. $5 b. $10 c. $15 d. $20

B

Erika decides to collect 30 pennies in the first bowl, 34 pennies in the second bowl, 38 pennies in the third bowl, and 46 pennies in the fifth bowl. How many pennies, do you think she will put in the seventh bowl? A. 50 B. 54 C. 58 D. 62

B

On day 1, Beth goes to the super market, and on day 2, she goes to the movies. For day , she decides to go to her friend's house. Then, on the fourth day she goes to the gym. If she continues her same schedule, where will she be going on the 30th day? A. Super Market B. Movies C. School D. Friend's House

B

During a play practice, one student is downstage and the other is upstage. Which action should the teacher take? A. Give microphone to the students to use. B. Have the downstage turn direction and face the character as well as the audien

B. Have the downstage turn direction and face the character as well as the audience

Over the past hundred years, human beings have been able to purchase inexpensive manufactured goods in much larger quantities than was possible in earlier times. Which of the following statements best describes the main reason for this development? a. Laws have been passed to regulate international trade and encourage investment in national economies. b. The energy stored in fossil fuels like coal and oil has been used to do more work than humans and animals had been able to do on their own. c. The creation of department stores and shopping malls has provided a way for people to purchase many different products in one location. d. Increased educational opportunities have made it possible for more people to accumulate wealth.

B

The following steps are taken to solve for x in the inequality: 5x + 3 < -18 + 10x Step 1: -5x + 3 < -18 Step 2: -5x < -25 Step 3: x < 25/5 Step 4: x < 5 Which of the step was performed incorrectly? A. Step 1 B. Step 2 C. Step 3 D. Step 4

B

Twenty-seven cubic centimeters is equivalent to how many liters? a. 0.03 liters b. 0.027 liters c. 0.003 liters d. 0.0027 liters

B

Which of the following is the most serious environmental effect of clear-cut logging in hilly regions of Michigan? a. replacement of forests with grassland ecosystems b. increased soil erosion and silting of local streams c. explosion of insect and pathogen populations in the slash left behind d. reduced diversity of tree species when the forest regenerates

B

Which of the following steps would the Federal Reserve most likely take during a period of high unemployment when prices are relatively stable? a. increasing the margin requirements on stock purchases b. reducing the discount rate on loans to banks c. requiring banks to increase the percentage of deposits held in reserve d. reducing the amount of paper currency in circulation

B

Stacy is a 3-year-old who is capable of reading and gleaning simple information from 2nd-grade books, although her oral reading is slow and somewhat labored. What stage of reading fluency has she achieved? A. pre-reading B. initial reading C. fluency D. reading to learn

B Although Stacy is only 3 years old, the fact that she can read and understand elementary-level books indicates that she has surpassed the pre-reading stage. Hence, option A is incorrect. At the same time, it is clear that she is still sounding out words and, thus, options C and D are incorrect too. Option B is the correct answer. Although it is unusual for 3-year-olds to be so advanced, there is considerable variability in the development of reading fluency. Stacy is an extremely precocious child who has already reached the initial reading stage—but not a higher stage, because she is still decoding words one by one.

Which of the following lists reflects an appropriate order for the introduction of written vocabulary to kindergarteners as they first begin to read? A. match, mad, map, man B. rat, run, red, runner C. pillow, pot, pear, pond D. man, mango, apple, cat

B Option A is incorrect, because the first word contains a consonant cluster. Option C is incorrect, because the first word is multisyllabic and begins with a stop sound. Option D is incorrect, because the first word contains two visually similar consonants, and because the second word is multisyllabic and may be an unfamiliar term for some children. Option B is the best answer. The first three words are simple consonant-vowel-consonant combinations that are monosyllabic, begin with continuous sounds, and represent words that are familiar to children in oral language. The fourth word builds on the first one in the list.

Which of the following types of writing would most directly help 2nd graders take the perspective of someone from a very different culture? A. learning log B. simulated journal C. response journal D. dialogue journal

B Options A, C, and D could help students take someone else's perspective. A learning log could be used to describe an inhabitant of a different culture, while a response journal could record the student's thoughts and feelings about that person. The exchanges in a dialogue journal could also focus on the life of an individual in a distant culture. However, option B is the best answer, because a simulated journal would require the student to take the other person's perspective throughout the entire writing process

Which of the following would be most helpful in developing phonemic awareness among preschoolers? A. During circle time, the teacher reads a list of words and asks children to clap each time they hear the name of an animal, but not when they hear the name of a food. B. During shared reading of a story about a snake, children are told that the first letter of the word "snake" is an "s," and they are asked to make an /sssss/ sound each time they hear the word. C. During show-and-tell, the teacher writes the name of each object on the board and pronounces it several times for the class. D. On the first day of class, during circle time, children take turns saying their own names, and then saying the first letter of their names.

B Phonological awareness activities help children notice the sounds that make up words. Option A is incorrect, because the only phonological skill required is auditory discrimination of the differences between animal names. Option C is incorrect as well. By writing the names of each object on the board, the teacher demonstrates correspondences between spoken and written language, but in itself does not contribute to phonemic awareness. Option D is inappropriate, because on the first day of preschool, at least some children will not know the first letter of their name. Option B is correct. By telling children that "snake" begins with an "s," and then asking them to make an /sssss/ sound each time they hear "snake," children can become better at segmenting that particular word by identifying its initial phoneme.

A 2nd-grade class is asked to read a famous nursery rhyme: "Little Miss Muffet / sat on a tuffet / eating her curds and whey. / Along came a spider / who sat down beside her / scaring Miss Muffet away." After reading the nursery rhyme, the teacher asks students what Miss Muffet sat on. Then, the teacher asks the class what they think a "tuffet" is. What sort of comprehension is the teacher's second question intended to tap into? A. literal B. inferential C. evaluative D. all of the above

B The literal meaning of a tuffet is neither given in the nursery rhyme nor familiar to contemporary American children and, thus, option A is incorrect. Because the question is about the likely meaning of a word, option C is also incorrect. Because options A and C are incorrect, option D could not be correct, either. Option B is the best answer. The teacher's question encourages children to make a simple inference based on the semantic and syntactic context established by the first two lines.

Which of the following countries are divided by religious lines? A. North Korea and South Korea B. India and Pakistan

B. India and Pakistan North and South Korea are separated due to war, not religion.

What is the study of biology and geology? A. oceanology B. paleontology C. zoology D. geography

B. paleontology Paleontology lies between biology and geology since it focuses on the record of past life, but its main source of evidence is fossils in rocks

A vending machine contains 12 types of snacks: some snacks are salty and some snacks are sweet. The ratio of sweet snacks to salty snacks is 1:3. How many types of sweet snacks are in the vending machine? a. 1 b. 3 c. 4 d. 12

B; 3

Which of the following activities would be the best way for a teacher to address the theme of courage in a civics class? a. taking a class trip to help clean up the local park b. discussing the activism of Dr. Martin Luther King Jr. c. establishing a classroom lost-and-found d. holding a mock trial to explain the concept of jury duty

B; Dr. Martin Luther King Jr. fought for the civil rights of people of color even though it was dangerous to do so. His story is a clear example of courageously doing the right thing, even when it might be unpopular or dangerous.

A teacher reads a biography about Harriet Tubman, a slave who helped others escape to freedom via the Underground Railroad. Which of the following would be most relevant to have students consider in a discussion about citizenship? a. honesty b. courage c. responsibility d. love

B; Harriet Tubman showed extreme courage when she returned to the South at great personal risk to help enslaved people escape.

Which of the following ideas from Thomas Hobbes influenced the US Constitution? a. Individual rights should take priority over collective rights. b. People should give up some of their rights and form a government to ensure order in society. c. Communism is the only way to govern people. d. People should create a utopia to avoid the evils of government.

B; Hobbes advocated for governments to create and ensure social order.

Before students begin a new literature book, Ms. Bean introduces them to a list of vocabulary words from the book and has them work in groups to develop skits that demonstrate the meaning of each word. Which of the following is Ms. Bean demonstrating? a. thinking aloud b. scaffolding c. identification of signal words d. informed judgment

B; Ms. Bean is scaffolding instruction by pre-teaching difficult vocabulary so that meanings are easily accessed during reading. In this way, the teacher can focus student attention on another reading comprehension strategy (beyond vocabulary development) during reading.

Which tool measures the volume of an object? a. thermometer b. graduated cylinder c. balance d. barometer

B; a graduated cylinder measures volume.

Ms. Jules has read and discussed several alliterative poems with her students. As a follow-up assignment, each student will write and illustrate an alliterative sentence for a class book. Which prewriting activity will best help student prepare for the assignment? a. drawing a concept map b. making a list of words c. developing a story map d. planning an outline

B; a list of words that begin with the same letter can be used as a reference when writing an alliterative sentence.

A teacher notices that several students are having difficulty determining the text structure of an informational article. Which strategy will help students achieve understanding? a. repeated reading of the text b. a mini-lesson on signal words c. a Venn diagram graphic organizer d. modeling thinking aloud

B; a mini-lesson on signal words would best help students gain a better understanding of how to identify text structure.

While reading the first drafts from a writing assignment, Mrs. Hamm notices that her students have a poor grasp of the proper use of commas. Which strategy would best serve Mrs. Hamm's goal of improving her students' understanding of comma usage? a. peer editing b. a mini-lesson c. a word investigation d. rewriting

B; a mini-lesson on using commas correctly would best serve the needs of the group and fill in the gaps of missing information.

Which of the following is considered a reliable source for research about California? a. a personal blog about living in California b. a research paper published by the State of California c. an advertisement for California real estate d. a letter to the editor about California roadways

B; a published study by a government institution is a reliable research source.

Which prewriting assignment would best prepare students for writing a quatrain poem? a. an investigation of the word quatrain b. listing pairs of rhyming words c. a mini-lesson on syllabication d. a concept map of the topic

B; a quatrain poem contains one or more four-line stanzas with a rhyme scheme, so a list of rhyming words is a helpful prewriting assignment.

An English-language learner who attempts to write simple sentences but uses a very limited vocabulary is functioning at which of the following language proficiency levels? a. L1 b. L2 c. L3 d. L4

B; an English-language learner at the L2 stage is at the very beginning stages of reading and writing and has a very limited vocabulary.

Which of the following is true of qualitative measures of text complexity? a. They are readability scores based on word frequency and sentence length. b. They are analytical measurements determined by knowledge demands. c. They are statistical measurements determined by computer algorithms. d. They are determinations of reading level based on professional judgment.

B; analysis of the knowledge demands required by a text is a qualitative measure of text complexity.

Antitrust law strengthens market forces to prevent monopolies. Which of the following terms best describes the implementation of antitrust law? a. social regulation b. economic regulation c. health regulation d. social security

B; antitrust law breaks up monopolies to allow fair competition within the economy.

A group of students has just finished reading an informational text about butterflies. How might the teacher extend learning on the topic? a. by having students repeat read the text b. by assigning each student a type of butterfly to research c. by explaining difficult vocabulary in the text d. by reviewing the text features

B; assigning a related research project extends learning on the topic after reading.

Ms. Frisillo notates the progress of her kindergarten students by indicating the date at which a child is able to master skills such as writing his or her name, identifying basic shapes, and memorizing his or her phone number. Which assessment tool is Ms. Frisillo using? a. rubric b. checklist c. anecdotal notes d. portfolio

B; checklists outline student performance criteria that teachers mark as students show mastery or each required skill.

A teacher has just completed a lesson on similes, and her students are lining up to go to recess. Which formative assessment strategy will give the teacher a quick rundown of how well students understood the lesson? a. a rubric b. an exit ticket c. a mini-lesson d. anecdotal notes

B; exit tickets can be turned in by students on their way out the door and quickly scanned for evidence of student understanding.

What is finger spelling? a. making crayon rubbings of sandpaper letters b. using the hands to sign letters c. counting letters in words using the fingers d. writing letters in trays of colored sand

B; finger spelling is a form of sign language that uses the hands to represent letters.

Which organism is an amphibian? a. snake b. frog c. dolphin d. pelican

B; frogs are amphibians.

Which of the following is NOT an appropriate example to use in describing a cause or effect of the US Civil War? a. inflation grew as more money was printed in both the North and the South. b. European immigrants were attracted to the South because of its booming economy, exacerbating tensions that led to the Civil War. c. Cotton exports to England stopped because the Union enforced a naval blockade of the South. d. Several states seceded from the Union when Abraham Lincoln became president.

B; immigrants from Europe actually preferred the North because there were more job opportunities there.

Which best states the purpose of the drafting stage of the writing process? a. revising the story's first paragraph b. organizing idea in a story element chart c. drafting the exposition of the story d. sharing the story with a classmate

B; it is most beneficial for students to use graphic organizers to shape their ideas before writing.

Which of the following strategies is most beneficial for students who are at the beginning stage of writing a story? a. revising the story's first paragraph b. organizing ideas in a story element chart c. drafting the exposition of the story d. sharing the story with a classmate

B; it is most beneficial for students to use graphic organizers to shape their ideas before writing.

Which of the following is true when teaching students to evaluate sources? a. Students should only consider internet sources. b. Students need to determine the credibility and contemporaneity of the source. c. Students in the lower elementary grades should only look at secondary sources. d. Students can include biased resources in their research.

B; it is the researcher's responsibility to ensure credibility and contemporaneity of sources; it is essential for teachers to ensure that students understand this.

Which activity would be helpful in teaching science processing skills? a. a vocabulary worksheet on which students match each science process to its meaning b. having a classroom pet that students care for c. a science workbook page about an experiment d. writing an essay describing science process skills

B; keeping a classroom pet would help students develop science process skills

Which of the following events is recognized as the beginning of the American Civil War? a. the Battle of Palmito Ranch b. the Battle of Fort Sumter c. the Missouri Compromise d. the Battle of Yorktown

B; on April 12th, 1861, the Confederate army attacked the Union-controlled Fort Sumter in Charleston, South Carolina, triggering the US Civil War.

Which of the following events contributed to the United States' entry into World War II? a. Germany's unrestricted submarine warfare b. the attack on Pearl Harbor c. the Battle of Britain d. the Battle of the Bulge

B; on December 7th, 1941, Japan bombed Pearl Harbor, hoping to prevent the US from interfering with its intent to invade Southeast Asia and other territories such as Guam and Hong Kong.

A teacher asks "What word do we hear when we say 'seat' without the '/s/'?" and instructs students to say the word. Which strategy is the teacher using to build phoneme awareness? a. phoneme blending b. phoneme deletion c. phoneme segmentation d. phoneme substitution

B; phoneme deletion requires students to remove phonemes from words to make new words.

Pounds per second is an example of which of the following? a. a nonstandard unit of measure b. a customary unit of measure c. a metric unit of measure d. elapsed time

B; pounds per second is the customary unit of measure for mass flow rate.

Which of the following is an example of a cross-cultural comparison? a. A chef studies variations in a recipe depending on the diner's preference for sweets. b. A Thai husband learns different holiday traditions from his wife, who is Chinese. c. Students compare the prices of groceries in different cities. d. A career coach helps clients determine their goals.

B; reviewing different holiday celebrations exemplifies studying different cultures.

Mr. Diego the history teacher, will be giving a final test the day before grades are due for the semester. He needs an assessment that will cover all of the information that has been learned so far this year in a format that will be quick and easy to grade. Which format would work best for Mr. Diego? a. essay b. selected response c. conferences d. portfolios

B; selected response are easy to grade and cover a breadth of information.

Which of the following is NOT an example of how to create a rubric? a. Teachers should avoid using negative language. b. Some indication of progression in skill level is unnecessary. c. Teachers should include specific examples demonstrating learning. d. The purpose of the rubric must be clear.

B; showing a progression of levels is one of the most important aspects of a rubric.

Which of the following would be a good activity for teaching students about life cycles and the four stages of metamorphosis? a. Have students create a food chain. b. Use mealworms and follow their development. Have students observe and record the process. c. Have students draw an animal cell. d. Have students grow a bean.

B; students can follow the life cycle of the mealworms and witness the process of metamorphosis in real time.

Which of the following is an appropriate use of a portfolio for summative assessments? a. to use in student-teacher conferences throughout a unit b. to compile blueprints, sketches, and planning for a specific project in order to measure student achievement at the end of that project c. to collect examples of what students feel is their best work d. to include self-assessments throughout a unit

B; summative assessments evaluate a student on his or her knowledge of skills and content. Portfolios should include a wide variety of evidence to help a teacher grade student work.

Mrs. Brotherton provides students with pattern blocks and asks the students to tile their desks, leaving no gaps and no overlapping tiles. What concept are the students learning? a. rotations b. tesselations c. reflections d. symmetry

B; tessellations are patterns created through the tiling of polygons.

Which of the following is NOT one of the amendments in the Bill of Rights? a. Citizens have the right to bear arms. b. The Senate must have equal representation. c. Citizens have the power of eminent domain. d. The government cannot quarter troops in private homes.

B; the Bill of Rights does not determine representation in Congress.

Which of the following protects freedom of religion in the United States? a. separation of powers b. the First Amendment c. power of eminent domain d. popular sovereignty

B; the First Amendment protects freedom of religion as well as freedom of speech, assembly, the press, and the right of the people to petition the government.

Which of the following would be the best example to use in a lesson discussing push factors in migration? a. political stability b. war c. more employment opportunities d. less crime

B; the danger posed by war clearly illustrates why an individual or group would leave an area or country.

Which of the following instruments has the lowest pitch? a. piccolo b. double bass c. trumpet d. trombone

B; the double bass is part of the string family and is the lowest pitched instrument in an orchestra.

Which expression has only prime factors of 3, 5, and 11? a. 66 x 108 b. 15 x 99 c. 42 x 29 d. 28 x 350

B; the factors for 15 and 99 are 3, 5, and 11.

Which of the following emerged during World War I? a. penicillin b. aircraft carriers c. radar d. nuclear power

B; the first time an airplane flew from a ship was in 1912.

What is the value of the digit 4 in the number 654,123? a. one thousand b. four thousand c. ten thousand d. forty thousand

B; the number represents four thousand.

The "cl" in the word "clap" is an example of a. a syllable b. an onset c. a phoneme d. a rime

B; the onset of a syllable is the beginning consonant or consonant blend.

Which is the primary purpose of expository writing? a. to entertain b. to explain c. to convince d. to describe

B; the primary purpose of expository writing is to explain.

A kindergarten teacher puts students in pairs and gives each student a domino with dots on both ends. The teacher asks the students to count the number of dots and write the number on a whiteboard. The teacher then asks whose numbers are higher, whose are lower, and if any partners have equal numbers. The teacher is most likely helping the students understand a. how to count the dots on a domino b. how to identify and compare numbers c. how to construct an addition sentence d. how to write numbers

B; the students will identify and write down the correct number of dots. By discussing how many numbers of dots each student in the class has, the students are comparing numbers.

A third grade teacher shows students a physical map of the state in which they live. The teacher highlights the location of agricultural resources, natural water sources, major roads, and areas with dense population. The teacher is most likely teaching the students a. that people tend to build major roads to agricultural resources. b. the relationship between human settlement and physical features. c. the reason for the state's high agricultural output. d. that there are many ecosystems in the state, and they must be preserved.

B; the teacher is showing the influence of geographical features on human and physical systems. People will go to places that can sustain life, such as sources of food and water.

How many phonemes are in the word "chick?" a. 2 b. 3 c. 4 d. 5

B; the word "chick" is made up of three phonemes: /ch/ /i/ /ck/.

Students board a bus at 7:45 a.m. and arrive at school at 8:20 a.m. How long are the students on the bus?

B; there are 15 minutes between 7:45 a.m. and 8:00 a.m. and 20 minutes between 8:00 a.m. and 8:20 a.m.; 15 + 20 = 35 minutes.

According to Vygotsky, teaching occurs when a. students undergo natural cognitive developments. b. teachers provide students with the support needed to complete tasks they would otherwise be unable to finish on their own. c. students demonstrate mastery of a new skill or concept. d. teachers offer students the independence to explore tasks on their own.

B; this area of optimal learning is called the Zone of Proximal Development.

A lesson on job trends and average salaries of people who graduate college communicates which of the following concepts to students? a. the global marketplace b. generating wealth c. market economy d. supply and demand

B; this lesson can show students all the variables to consider as they contemplate earning money in the future.

The formula for distance is d = R x T. How long will it take a plane to fly 4,000 miles from Chicago to London if the plane flies at a constant rate of 500 mph? a. 20 hours b. 8 hours c. 45 hours d. 3.5 hours

B; time is distance divided by rate, 4000 mi/500 mph = 8 hours.

Which of the following would be the most appropriate graphic organizer for a student who is asked to organize a series of significant events? a. KWL chart b. timelines c. Venn diagram d. semantic web

B; timelines allow students to sequence events, organizing their thoughts.

A student is able to write the word "cat" by sounding out each letter. What concept is the student demonstrating? a. phonological awareness b. phonics c. syllabication d. word analysis

B; when a student connects oral sounds to written letters, he or she is demonstrating an understanding of phonics.

Which of the following is NOT a responsibility of the executive branch? a. approving laws b. making laws c. implementing laws d. enforcing laws

B; while the executive branch can approve them, only Congress can make laws.

Which of the following is an effective strategy for strengthening decoding skills? a. teaching character analysis b. teaching word families c. teaching active listening d. teaching fact and opinion

B; word family instruction is a decoding strategy that reinforces student understanding of word patterns.

If a metal rod is broken in half, how does it expand?

Broken in half or not, when a rod is heated, the atoms and molecules expand, causing the rod to expand as well

-3 < d e < 10 d < f < e From the information above, if d, e, and f are integers, which of the following could be the value of f? A. -8 B. -6 C. -2 D. 12

C

What is the perimeter of a pentagon where one side length is 2 inches? a. 4 inches b. 8 inches c. 10 inches d. 32 inches

C; add the length of all sides to find the perimeter.

Which of the following describes a significant effect of the depletion of the ozone layer? a. people get darker skin b. more UV rays enter the earth c. increased skin cancer rates, build-up of greenhouse gases, and increased UV levels d. increased build-up of greenhouse gases

C; all of these are consequences of the depletion of the ozone layer.

A class has p number of students. If z numbers of students play soccer, which expression shows the fraction of students who do NOT play soccer? A. (p + z)/Z B. (p - Z)/Z C. (p - z)/p D. (p + z)/p

C

A kindergarten teacher regularly leads classroom discussions before beginning a new read-aloud of informational texts. For example, he may conduct a book walk of the pictures, point out some key vocabulary, and then ask students to share what they already know about the subject of the book. This practice most clearly demonstrates the teacher's understanding of which of the following factors that influence emergent readers' construction of meaning? a. the role of setting a purpose and goal prior to reading a text b. the importance of using textual evidence to draw conclusions c. the role of activating background knowledge related to a text d. the importance of determining a text's main idea and details

C

A second-grade teacher would like students to apply the concept of harmony during a music lesson. Which of the following activities would be most appropriate for this purpose? a. listening to a major scale b. playing a rhythm on percussion instruments c. singing a round in groups d. moving fast and slowly to musical recordings

C

A sixth-grade teacher is planning a project in which students will use disposable cameras to document their lives outside of school. The students' photographs will be featured in an exhibit at the public library. The teacher would like to provide students with appropriate guidance throughout the project. Which of the following steps should the teacher take first? a. explaining to students how they can upload their photographs onto classroom computers for editing b. showing students a variety of photographs by professional portrait photographers and photojournalists c. reviewing privacy laws and providing release forms to be signed by the individuals that the students photograph d. demonstrating techniques for composing photographs and letting students practice the techniques in class

C

The Upper Peninsula of Michigan has the second highest average snowfall of any nonmountainous region of the world. Which of the following best explains the high average yearly snowfall in this area? a. Cold air masses coming down from the north contain large amounts of moisture that falls as snow when they meet warm air masses coming up from the south. b. Jet stream winds pick up large amounts of moisture from the interior of the continent and drop it as snow when they encounter cold air above the Great Lakes. c. Cold northern winds pick up large amounts of moisture as they blow over the relatively warm surface of Lake Superior and drop it as snow when they reach the colder shore. d. Dense coniferous forests covering the Upper Peninsula increase absorption of solar radiation, which warms the surface and causes moisture in cold air masses from the north to fall as snow.

C

The information below best describes major characteristics of which of the following world regions? • The region is bounded by deserts, mountains, and the ocean. • The region has 3 percent of the world's land area and 23 percent of the world's population. • Major religions of the region include Islam, Buddhism, and Hinduism. a. North Africa b. sub-Saharan Africa c. South Asia d. Southeast Asia

C

Assuming that a kindergarten class already has alphabetic recognition for six high-frequency consonants, what should the teacher do next? A. Teach children how to recognize low-frequency consonants, such as "q" and "z." B. Continue teaching high-frequency consonants and make a "letter list" of consonants already learned that can be posted at the front of the room. C. Teach children high-frequency vowels so that they can begin to read simple words in picture books. D. Have children make drawings of the consonants and vowels that they already know, and then post the drawings around the room.

C All of the options would promote literacy development. However, it is important that literacy instruction takes place in a meaningful context. The focus should not be on isolated skills but rather on the application of these skills to meaningful, interesting activities. As children acquire letter names and alphabetic recognition skills, they should be given the opportunity to apply these skills to reading as soon as possible. Options A, B, and D all reinforce children's understanding of individual letters. In contrast, option C would allow children to apply their understanding of individual letters to the task of reading actual words. Hence, option C is the best answer

Which poetry writing assignment would best reinforce syllable concepts? a. a diamante poem b. an acrostic poem c. a haiku poem d. a shape poem

C; an understanding of syllables is necessary for writing a haiku poem.

During the first week of school, a pre-kindergarten teacher asks each child to bring something blue from home to talk about in a show-and-tell format. What is the main purpose of this activity? A. to strengthen children's pronunciation skills B. to provide new vocabulary words for the class to learn C. to help children become comfortable about talking in front of the group D. to support the development of critical listening

C Any use of oral language allows children to exercise their pronunciation skills. Asking children to discuss something they have brought from home does not help pronunciation more than other ordinary uses of oral language. Hence, option A would not be the main purpose of the activity. For the same reason, option B would not be the main purpose, because there are so many opportunities, both planned and unplanned, for teaching new vocabulary. Pre-kindergarten teachers would probably not emphasize critical-listening skills so early in the school year, so option D would not be the main purpose either. Option C is the best answer. During the early weeks of school, it is important for the pre-kindergarten teacher to help children become comfortable with expressing themselves in front of their classmates. In this example, the teacher has chosen an activity that bridges the gap between school and home and also allows children to discuss something familiar to them.

Which of the following would be most effective in fostering print awareness among preschoolers? A. a guided reading activity in which children practice identifying the main point in a passage B. an invented spelling activity, in which children are encouraged to write a story (children are told they can write what they want and use any spelling they prefer) C. a book making activity, in which each member of the class draws a picture on a "page;" the pages are then combined and jointly "read" by teachers and students D. a choral reading activity based on a very simple text

C Options A and D are incorrect, because guided and choral reading are too advanced for preschoolers. Option B is incorrect for a similar reason. Some preschoolers engage in invented spelling and in spontaneously doing so, they exercise their emerging knowledge of print. But not all children are capable of making letters by preschool age and, thus, asking them to write a story may result in confusion. Option C is correct. By combining pages in a book and reading each page in succession, the teacher reinforces some basic conventions of print, such as the idea that books contain pages, the pages of books contain information, and readers examine pages in a left-to-right sequence.

In order to promote reading fluency, which of the following is the best approach for introducing a 2nd-grade class to a play that has been written for their grade level? A. paired reading B. Sustained Silent Reading (SSR) C. Oral Recitation Lesson (ORL) D. repeated reading

C Options A, B, and D all represent useful approaches to the promotion of reading fluency. However, an Oral Recitation Lesson is probably best for introducing a new text to this age group. The ORL would give students guidance in the oral reading of the play, because the teacher would model fluent reading, and then discuss semantic and prosodic issues of pertinence, before students read the play out loud. Thus, option C is the best answer. The other options reflect methods that would be more effectively used after the ORL had been implemented.

In a unit on Texas history, Mrs. Jones asks her 4th graders to work in groups to create written descriptions of the social, military, and political importance of the Alamo. Once each of the groups has generated some ideas to work from, which of the following would be a good strategy for proceeding? A. freewriting B. brainstorming C. revising D. drafting

C Options A, B, and D are useful for generating ideas, but they are not designed to refine existing ideas. Option C is the best answer. Once students have ideas to work from, revision is the next step if the ultimate goal is a finished product that will be shared with others

After reading "Miss Muffet" and discussing what a "tuffet" may be, the teacher asks students why they suppose the author of the rhyme used such an unusual word. The teacher then leads a class discussion of this question. To help fuel discussion, the teacher intends to inform students that the rhyme was created in England a long time ago. The teacher also intends to say something about the process of creating rhymes. What sort of comprehension is the teacher trying to stimulate through class discussion? A. literal B. inferential C. evaluative D. all of the above

C Students have not been given a direct answer to the teacher's question, so option A is incorrect. If option A is incorrect, option D must be incorrect too. Although one can infer from the teacher's comment that the word "tuffet" reflects the particular time and place of the rhyme's creation, inference is not the sole focus of the class discussion and, thus, option B is not the best answer. That is, option B is not the best answer because it is only partially true. Option C is the best answer. In order to arrive at some sense of why the word "tuffet" was used, students will have to explore the possibility that English speakers from different times and places use different vocabulary, and they will have to consider the role of end rhyme in poetry. Looking at the poem from historical and literary perspectives reflects evaluative comprehension

Micah invites 23 friends to his house and is having pizza for dinner. Each pizza feeds approximately four people. Micah does not want a lot of leftovers. How many pizzas should he order? a. 2 b. 5 c. 6 d. 7

C; 23 people / 6 pizzas = 3.8 people per pizza. There would be some pizza left, but this is the closest Micah will be able to get to 4 people per pizza.

Simplify (5^2 - 2)^2 + 3^3. a. 25 b. 30 c. 556 d. 538

C; 529 + 27 = 556.

Which of the following was responsible for the New Deal? a. Herbert Hoover b. Rutherford Hayes c. Franklin D. Roosevelt d. Jimmy Carter

C; Franklin D. Roosevelt supported laws that helped those unemployed and impoverished due to the Great Depression.

Ken has 6 grades in English class. Each grade is worth 100 points. Ken has a 92% average in English. If Ken's first 5 grades are 90, 100, 95, 83, and 87, what did Ken make for the 6th grade? a. 80 b. 92 c. 97 d. 100

C; If Ken has 6 scores that average 92%, his total number of points earned is found by multiplying 92 x 6 = 552. Adding up all of the other scores, the remaining score is 97.

Which of the following historical figures was a major architect of the US Constitution? a. Thomas Jefferson b. George Washington c. James Madison d. Patrick Henry

C; Madison not only advocated for the ratification of the Constitution, he had also helped to write the Federalist Papers.

Which assessment is a type of formative assessment? a. standardized test b. benchmark test c. exit ticket d. end-of-unit assessment

C; a ticket out the door is an informal, formative assessment.

Mr. Clayborne wants to gather information about Charity's progress in reading to help him target her small group instruction. Which assessment is most appropriate for him to use? a. state standardized test b. aptitude test c. anecdotal records d. intelligence test

C; anecdotal records are an informal assessment that helps guide instruction.

Which of the following is most likely a lasting influence ancient Romans had on modern society? a. the development of direct democracy b. the usage of columns in architecture c. the development of republican democracy d. literacy

C; before it was an empire, Rome was a republic led by senators elected to the Senate.

Amanda is having difficulty adding fractions with unlike denominators. With which of the following does Amanda need support? a. how to simplify exponents b. how to multiply decimals c. how to find the least common multiple d. how to solve a multi-step equation

C; being able to find the least common multiple helps a student find an equivalent fraction with a common denominator.

Which of the following is an appropriate topic in teaching students about decisions surrounding substance abuse? a. understanding the physical consequences of substance abuse b. knowledge of alcohol, tobacco, and illegal drugs c. A and B d. none of the above

C; both A and B are equally important when teaching students about substance abuse so that they can make intelligent decisions.

Mr. Falls is reading aloud from a predictable picture book. he has written a refrain from the story on a sentence strip that is displayed on a pocket chart. Each time the refrain occurs in the story, Mr. Fall signals his students to read aloud with him as he points to each word. Which of the following is Mr. Falls demonstrating? a. decoding b. predicting c. concept of print d. word analysis

C; by having students listen to predictable text and chant the refrain with him as he points to individual words, Mr. Falls is reinforcing concepts of print, clarifying that words and letters convey meaning, words are constructed of letters, and test is read from left to right.

Which of the following is a responsibility of a citizen of the United States? a. to treat others with kindness b. to join after-school clubs c. the suffer the consequences of breaking a law d. to start a business

C; citizens have a responsibility to understand laws and pay the consequences if they break them.

Which type of chemical reaction takes place when kerosene reacts with oxygen to light a lamp? a. oxidation b. neutralization c. combustion d. convection

C; combustion is a chemical reaction that produces carbon dioxide and water. Burning lamp oil is combustion.

Mrs. Franco's students meet with her individually so that she can ask them questions about what they have learned. Which assessment format is Mrs. Franco using? a. performance b. observation c. conference d. portfolio

C; conferences are meetings between teacher and student in which learning is orally assessed and evaluated.

Which strategy would best help Mr. Green develop his students' reading vocabularies? a. having students peer edit each other's short stories b. conducting a mini-lesson on using a dictionary c. providing weekly opportunities for word investigations d. brainstorming a list of words in a particular category

C; consistently providing opportunities for students to investigate words in-depth increases the probability that they will retain comprehension of the words over time.

Mrs. Omeira's class wants to find the probability of pulling a red gumball out of a jar that contains 50 gumballs. What information do the students need to know to find the answer? a. How many green gumballs are in the jar? b. How many different colors of gumballs are in the jar? c. How many red gumballs are in the jar? d. How long have the gumballs been in the jar?

C; it is important to know how many red gumballs there are since the class already knows how many gumballs there are in total.

Which of the following is NOT a positive nonverbal clue in active listening? a. smiling at the speaker b. leaning forward slightly c. looking down at the floor d. sitting up straight

C; looking down at the floor does not demonstrate active listening because one's focus is not on the speaker.

An ice chest contains 24 sodas, some regular and some diet. The ratio of diet soda to regular soda is 1:3. How many regular sodas are there in the ice chest? a. 1 b. 3 c. 18 d. 24

C; one out of every four cans is diet, so there is one diet can for every three regular cans.

Mr. Couch has his students take turns doing show-and-tell presentations every Friday afternoon. Which of the following is a primary benefit of show-and-tell opportunities for students? a. Students make decisions about what to share b. Students rest at the end of a busy week c. Students practice speaking in front of an audience d. Students are rewarded for doing their homework

C; opportunities for show-and-tell provide students with practice speaking in front of an audience. The audience members also receive practice exercising their active listening skills.

Students have been placed in small groups to prepare reader's theater presentations that they will perform for each other. How might each group best meet the needs of its audience? a. by reciting their lines in quiet voices during the performance b. by passing around a script to read from during the performance c. by using classroom materials as props during the performance d. by skipping some parts of the story during the performance

C; props add interesting visuals to a performance and reinforce comprehension.

Which of the following would be a good introduction to a unit on astronomy? a. to read the overview of the chapter on astronomy in the science textbook b. to take a pretest about astronomy c. to show students videos about the solar system d. to have students draw what they think the solar system looks like

C; showing videos about a subject is a great hook to capture students' attention and create a level of excitement and enthusiasm.

Each spring, the state gives a multiple choice test to every student in the first week of April using explicit instructions regarding security, time, and monitoring. Which type of test is described? a. a formative assessment b. an anecdotal record c. a standardized test d. remediation

C; standardized tests are formal assessments given to students in a specific way.

Which step should students take before making a hypothesis in a scientific experiment? a. interpret data b. make a graph c. research d. do the experiment

C; students need to conduct research before making a reasonable and testable hypothesis.

Which of the following is NOT an example of human geography? a. studying the importance of trade among Central American countries b. examining regional differences in cuisine around the world c. studying the distribution of fauna and flora in North America d. deciding whether to build retail outlets based on local population

C; studying only flora and fauna is physical geography; the geographer is not studying human interactions with or impact on the earth.

Mr. Ferguson's students are taking an end-of-course exam that they must pass as a graduation requirement. Which type of assessment are they taking? a. diagnostic assessment b. formative assessment c. summative assessment d. intelligence test

C; summative assessments evaluate what students have learned.

How are whiteboards useful as formative assessment tools? a. They can be used by students to practice penmanship skills. b. They can be used by students as exit tickets to show understanding of a concept. c. They can be used by students to demonstrate understanding at a glance. d. They can be used by students as a self-assessment tool

C; teachers can make at-a-glance assessments of understanding by having students simultaneously show answers to a question on whiteboards.

Which of the following amendments prevents the government from quartering troops in private homes? a. the Seventh Amendment b. the Fourth Amendment c. the Third Amendment d. The Ninth Amendment

C; the Third Amendment forbids the government from quartering troops in homes, an abuse suffered by Americans under British rule.

Which of the following is a primary reason to discuss needs and wants with students? a. Students must understand how to budget for their wants. b. Students can use this concept to understand opportunity cost. c. Needs and wants impact supply and demand. d. Understanding needs and wants is essential to understanding all other elements of economics.

C; the concept of supply and demand is fundamental in understanding the free market economy.

Which of the following best describes an independent variable? a. the amount of paint a painter needs to paint a room b. the number of errors a student gets on a test c. the dosage and timing of an anti-inflammatory drug to test its impact d. how much time it will take to drive between Houston and San Antonio

C; the doctor or scientist who carries out the experiment controls the timing and dose.

A teacher says "hat" and instructs students to produce the sounds they hear in the word. Which strategy is the teacher using to build phoneme awareness? a. phoneme blending b. phoneme deletion c. phoneme segmentation d. phoneme substitution

C; the strategy of phoneme segmentation requires students to separate the phonemes in a word.

A second grade teacher has given students manipulatives and whiteboards. The teacher writes the following equation on the board; 12 + 5 = 9 + ? and has students write and solve the problem. Which algebraic concept is the teacher assessing? a. patterning b. variables c. equivalency of numbers d. graphing

C; the students are being asked to find the number that will make a number sentence equivalent, or the same, as another number sentence.

Mrs. Trent is preparing a peer-editing document for her students. What should she include? a. a list of criteria for checking mechanics and a section for rewriting parts of the text under review b. a section for compliments, a section for suggestions, and a section for criticisms c. a list of criteria for checking mechanics, a section for helpful suggestions, and a section for compliments d. a section for writing tips, a section for compliments, and a section for comments

C; this is a common three-part strategy used for peer editing.

Letter and Word Concepts

Children learn◦ to identify letter names & upper and lowercase letters◦ that the first word of a sentence is capitalized

Cognitive Approach

Children must develop cognitive skills before they speak.

What are students measuring when a teacher gives them a hulla hoop and wants them to put a dot on one spot of the hulla hoop and then roll it across the floor?

Circumference

What was the greatest influence in the 20th century on U.S. Civil War and Reconstruction?

Civil rights movement

A boy wants to know more about his population. What can he use?

Consensus Data- Process of collecting info about every member of a population.

Understanding the word hairless

Considering the context in which the word is used

What can happen during any stage of reading development?

Contextual Analysis

4, 10, 16, 22, 28, 24, 20, 16, 12, 8, 4, 10, 16 If the above pattern continues, what will be the 16th number in the sequence? A. 16 B. 22 C. 28 D. 24

D

A scientist would need both a graduated cylinder partially filled with water and an electronic scale to determine the: a. mass of a rock sample. b. surface area of a flight feather. c. volume of a plant oil sample. d. density of a bone.

D

Eighth-grade students are co-hosting a field day at the adjacent elementary school. With help and guidance from two physical education teachers, small groups of eighth graders create individual challenge stations and group games that they will facilitate during the elementary school's field day. This activity is particularly appropriate for promoting the eighth-grade students........ a. understanding of the developmental stages of elementary students. b. awareness of physical and motor differences related to age. c. ability to adapt to changing group dynamics. d. responsibility and leadership skills.

D

In which of the following situations would a box-and-whiskers graph be the most appropriate visual representation to use to display data? a. An homeowner wants to compare the amount of money spent on individual household expenses in a particular month to the total monthly expenses. b. A business owner wants to see trends in sales data over a 24-month period. c. A bookstore employee wants to compare the numbers of each type of book sold in the previous two months. d. An environmentalist wants display several statistics to summarize a large number of pH readings from a stream.

D

Which of the following events or developments had the greatest impact on economic growth in the Midwest region of the United States during the first half of the nineteenth century? a. the Louisiana Purchase b. the construction of the National Road c. the War of 1812 d. the opening of the Erie Canal

D

Which of the following factors is the main cause of global economic interdependence? a. growing world concern about the provision of energy sources b. the decolonization of most parts of Asia, Africa, and Latin America c. increased population movement among countries d. the international reduction of barriers to trade and investment

D

Which of the following scenarios would provide the best evidence that a kindergarten student is progressing according to typical stages of literacy development? a. Now the student provides detailed drawings to accompany his written symbols, whereas he never illustrated his writings at the beginning of the school year. b. At the beginning of the school year, the student often incorporated words into his writings that he had copied from environmental print, but now he only uses his own original symbols. c. Now the student knows that he is not writing real words, whereas at the beginning of the school year he would point to clusters of letters and call them words. d. At the beginning of the school year, the student told a different story each time he reread his writings, but now he follows the letters with his finger and uses similar words when rereading.

D

Which of the following teacher actions during a read-aloud of a "big book" is likely to be most effective in supporting kindergartners' vocabulary development? a. reading dramatically using exaggerated intonation and placing extra emphasis on key words in the text b. running a pointer under the text while reading and pausing to have students repeat key words from the text several times chorally c. writing a list of key words from the text on the board and referring students to the list as each word appears during the reading d. pausing regularly to elaborate on the content of a passage and to question students about key words in the text

D

Which of the following can a parent do to help a 4th grader research a class project on whales? A. Encourage the child to use the school library and to ask the librarian for assistance. B. Help the child use the Internet as a resource. C. Help the child decide which index selections of a student encyclopedia would be most relevant. D. all of the above

D A is a correct option. Although the parent would not be directly helping the child, encouraging the child to use the library and seek expert assistance would be beneficial. B and C are correct options, because in each case, the parent would be providing expert guidance. Hence, option D is the best answer

A teacher recites "Peter Piper" very slowly for her kindergarten class. Afterward, she asks the class which sound they heard a lot of. What aspect of language is the teacher trying to sensitize her class to? A. rime B. blending C. phonology D. alliteration

D Option A is incorrect, because rime consists of the vowel(s) and consonant(s) that follow an onset sound. The rimes in "Peter Piper" tend to be different from word to word. Option B is incorrect, because children are not given the opportunity to hear or engage in blending. Option C is incorrect, because it is much too general. Option D is the correct answer. "Peter Piper" is extremely alliterative, owing to the numerous repetitions of the /p/ sound.

A kindergarten teacher encourages his class to tell stories from wordless books using narrative and role-play. What skill does this activity nurture? A. phonological awareness B. print awareness C. graphophonemic knowledge D. emergent literacy

D Option A is incorrect, because this activity provides no particular exercise for the ability to notice sounds in words. Likewise, option C is incorrect, because the activity does not call children's attention to letter-sound correspondences. Although the activity does reinforce the understanding that books contain stories, the teacher uses wordless books and, thus, option B is incorrect. The correct answer is option D. By looking at the pictures in wordless books and both describing and acting out a sequence of events, children have an opportunity to practice their narrative skills and link an unfolding narrative to a sequence of pages in a

There are many types of clouds. Out of the 4-main types of clouds, which clouds extend through all height levels, rain, storm? A. Stratus B. Cirrus C. Cumulus D. Nimbus

D. Nimbus

Which of the following factors most increased cultural diversity within the United States during the early nineteenth century? A. The relocation of American Indians in western territories B. The arrival of European immigrants in urban areas C. The migration of African Americans to northern cities D. The arrival of Asian workers to build railroads

D. The arrival of European immigrants in urban areas European immigrants represented many cultures, so they increased diversity in the United States more than any action by a single group.

What weight do you measure field of mice in? A. pounds B. ounces C. kilograms D. grams

D. grams 1 gram is about the mass of a metal paperclip. A pencil and an empty soda can each weigh about 0.5 oz., and a tennis ball weighs about 2 oz. Field mice are tiny and weigh around 15-25 grams. This is not even 1-ounce

What is the value of 3x + 7x if x = 8? a. 10 b. 21 c. 31 d. 80

D; 3(8) + 7(8) = 24 + 56 = 80.

Which expression is equivalent to dividing 400 by 16? a. 2(200 - 8) b. (400 / 4) / 12 c. (216 / 8) + (184 / 8) d. (216 / 16) + (184 / 16)

D; 400 / 16 = 25.

Which of the following continents is located in both the Eastern and Western hemispheres? a. Europe b. Africa c. North America d. A & B

D; Europe and Africa are located in both the Eastern and Western Hemispheres.

Which is an example of a secondary source? a. a handwritten letter by a former president b. a photograph from the turn of the century c. an audio recording of a jazz composition d. a book that discusses a historical time period

D; a book that discussed a historical time period is an example of a secondary source because it's written by an author who synthesizes and analyzes primary sources to form conclusions.

Which of the following can be classified as narrative writing? a. an opinion piece on a political candidate b. an essay on the causes and effects of erosion c. a poem that evokes the feeling of a spring thunderstorm d. a funny story about an adventure at the zoo

D; a personal story with a plot arc is an example of narrative writing.

Which of the following is an example of the separation of powers? a. No members of Congress can serve in another branch of government b. The president cannot vote on legislation c. checks and balances d. all of the above

D; all choices are different examples of the separation of powers.

Which strategy is most typically used to grab the audience's attention at the beginning of an oral presentation? a. eye contact b. a statement of topic c. a loud voice d. an engaging hook

D; an engaging hook prior to the introduction of an oral presentation is typically used to grab the audience's attention.

Making a judgment about a piece of artwork is called a. visual communication b. aesthetics c. debate d. art critcism

D; art criticism includes the evaluation of a piece of art following its analysis.

Which activity is most appropriate for a student who is learning to count to ten? a. comparing integers on a number line b. drawing pictures of combinations that equal ten c. matching the numeral ten to cards with ten items on them d. counting, comparing and sorting ten plastic bears

D; counters and toys are concrete objects.

Which behaviors best demonstrate focusing on a speaker during active listening? a. moving around and eye contact b. sitting up straight and interjecting questions c. frowning and crossed arms d. eye contact and leaning forward

D; eye contact and leaning forward demonstrate that the listener is actively focusing on the speaker's message.

A project in which students research and explore significant contributions made by their community is an example of a. questioning only b. collaboration c. data interpretation only d. inquiry-based learning

D; in inquiry-based learning, students gather relevant sources and interpret them in order to develop their own conclusions.

Which of the following factors might contribute to inflation? a. high interest rates and a low amount of printed currency b. high unemployment rates c. a decrease in supply and a low amount of printed currency d. a decrease in supply and a large amount of printed currency

D; inflation occurs when prices increase. Both of these factors contribute to inflation.

Which of the following objectives is most effective? a. Student will read three books about wild cats. b. Talk to students about different types of wild cats. c. Students will appreciate the diversity of wild cats that exist. d. Students will compare and contrast three different types of wild cats.

D; it's measurable and student-focused.

Which of the following concepts involves understanding how the forms and structures of words contribute to their meanings? a. phonology b. mechanics c. paraphrasing d. morphology

D; morphology investigates how the forms and structures of words contribute to their meanings.

Which of the following is NOT a method teachers should encourage students to use when conducting online research? a. selecting appropriate keywords b. looking through a website for relevant information c. checking the validity of the source by researching its author or publisher d. choosing the first search result as a valid source

D; not all search results are relevant or have correct information. Students should be encouraged to search until they can verify the validity of a source and determine if that source provides relevant information.

A teacher asks students to identify the word formed when the /r/ at the beginning of the word "rug" is replaced by /b/. Which strategy is the teacher using to build phoneme awareness? a. phoneme blending b. phoneme deletion c. phoneme segmentation d. phoneme substitution

D; phoneme substitution requires students to replace phonemes in words to make new words.

Which of the following is an example of an informal assessment? a. annual state testing b. college admissions tests c. IQ tests d. portfolio assessments

D; portfolios are only used by teachers to make instructional decisions.

A second-grader repeatedly throws wads of papers across the room. A teacher using positive behavior supports would respond in which of the following ways? a. The teacher reminds the student of the classroom rule: "No throwing objects." b. The teacher stops class to discipline the student. c. The teacher makes the student stand in the corner of the room for the remainder of the day. d. The teacher monitors the student to determine why he or she was throwing the paper.

D; positive behavior supports is based on the idea that all behavior is rational.

Robbie has a bag of treats that contains 5 pieces of gum, 7 pieces of taffy, and 8 pieces of chocolate. If Robbie reaches into the bag and randomly pulls out a treat, what is the probability that Robbie will get a piece of taffy? a. 1 b. 1/7 c. 5/8 d. 7/20

D; probability is the number of favorable events divided by the number of possible events. In this case, Robbie pulls out one treat from a bag that contains 7 pieces of taffy; therefore 7 is the number of favorable events. There are 20 total treats in the bag because 5 + 7 + 8 = 20.

Which of the following best describes reading rate? a. using appropriate vocal cues when reading aloud b. decoding words correctly when reading aloud c. having a significant inventory of known sight words d. reading smoothly and steadily when reading aloud

D; reading smoothly and steadily when reading aloud is an example of reading rate.

A sign of the freeway indicates that you are 150 miles from the city of Pittsburgh. This shows your a. absolute location. b. physical geography. c. geographic feature. d. relative location.

D; relative location refers to the location of a place relative to another, in this case Pittsburgh.

Which of the following is an example of a representative democracy? a. Citizens vote on legislation. b. Citizens vote on constitutional amendments. c. Citizens can directly vote for policies. d. The House must have proportional representation in order to adequately reflect population size.

D; representatives are elected to the House to represent the people; they vote on legislation on their behalf.

What are some factors teachers should consider when selecting songs for students at the elementary level? a. a manageable rhythmic pace b. repeating lyrics and melodies c. few vocal demands d. all of the above

D; rhythmic pace, repetition, and vocal demands are all equally important factors in choosing appropriate songs for elementary-level students.

Controversy over the Kansas-Nebraska Act of 1854 illustrated which of the following? a. immigration b. Reconstruction c. westward expansion d. sectionalism

D; the Kansas-Nebraska Act further divided the nation over slavery.

Which of the following letters is most likely to be introduced first in progressive phonics instruction? a. a b. g c. y d. m

D; the letter "m" is most likely to be introduced first because it contains its sound in its name and only forms one sound in words.

If the pattern continues, what is the next number? (2, 5, 14, 41, 122...) a. 244 b. 264 c. 422 d. 365

D; the pattern follows the equation 3n-1 = x. n = the last number in the set and x = the next number in the set.

Which number has a prime factorization of three odd numbers and one even number? a. 9 b. 21 c. 45 d. 90

D; the prime factorization of 90 is 2 x 3 x 3 x 5.

Which letter or word part forms the rime in the word "cake?" a. c b. e c. cak d. ake

D; the rime in "cake" is formed by all of the letters that follow the beginning consonant.

A teacher plays a game with students called "Mystery Bags." The students must feel an object in a bag and try to determine what it is without looking at it. Which science process skills would the students be using during this activity? a. measurement and classification b. communication and classification c. predicting and measurement d. observing and inferring

D; the students are making observations about the object based on what they feel inside the bag. Using these observations, the students can infer what the object is.

Read the sentence from "The Wonderful Wizard of Oz" by Frank Baum. "There now came a sharp whistling in the air from the south, and as they turned their eyes that way they saw ripples in the grass coming from that direction also." How would the tone of the sentence change if "sharp" were replaced by "gentle"? a. the tone of the sentence would remain the same. b. the tone of the sentence would become more negative. c. the tone of the sentence would be indistinguishable. d. the tone of the sentence would become more positive.

D; the use of "gentle" instead of "sharp" would change the tone of the sentence by making the storm seem less threatening and more playful.

A teacher asks students to compare different songs and poems about the War of 1812. In small groups, they must explain how these sources help people understand significant historical events and other people's perspectives on them. This assignment is an example of a. questioning b. chronology c. interpreting timelines d. historical analysis

D; this assignment requires students to understand the events of the War of 1812, analyzing primary resources, and then develop their own conclusions.

Which of the following happened to Jewish people under the Nazi regime? a. Their homes and businesses were attacked and destroyed. b. They were forced to live in ghettoes, overcrowded neighborhoods with poor living conditions. c. They were sent to concentration camps and murdered. d. all of the above

D; under the Nazis, Jewish people were forced to live in ghettoes; their businesses and homes were destroyed and they suffered other persecution. Jews, communists, Roma, homosexuals, and others were also forced into slave labor in concentration camps, and millions were murdered.

When introducing equations to students, which step should Ms. Martin take first? a. assign students to work with a partner to solve 3x + 5 = 20 on whiteboards b. assign students to work individually to solve 3x + 5 = 20 on paper c. assign students to work with a partner to draw a picture representing 3x + 5 = 20 d. assign students to use mats and counters to demonstrate 3x + 5 = 20

D; using concrete objects will help students obtain a conceptual understanding of the mathematical principles behind solving equations.

Which of the following activities would help students learn and understand how sound travels from one end to the other? a. making a flute out of straws b. putting a balloon over a plastic cup, pouring salt on it, and playing music c. using spoons to tap on jars filled with various amounts of water d. making a string phone

D; when students speak into the cups, they create sound waves that turn into vibrations.

Which of the following best defines decoding?

Decoding refers to the process of reading — translating words into sounds and ideas.

Earliest regions subtropics

Evaporation rates often exceed rainfall increasing the waters salinity

Kid wants to show process of making food at school visually, what kind of visual should he use?

Flow chart

Explain approaches to take for emergent readers?

For emergent readers, these are young students who are just beginning to learn to read, ect. Approaches include singing the alphabet and introducing print concepts during read alouds. Singing... Create an Inviting Reading Environment.... Take a Picture Walk. ... Become a Storyteller. ... Use Repetitive Texts. ... Picture Clues can Help with Unknown Words. ... Teach Everyday Words in Groups.

Newton's Third Law

For every action there is an equal and opposite reaction (ex. when a hammer strikes a nail the nail hits the hammer just as hard)

What is "Soft Power" in government?

In politics (and particularly in international politics), soft power is the ability to co-opt rather than coerce (contrast hard power). In other words, soft power involves shaping the preferences of others through appeal and attraction.

What happens when a magnet is cut in half?

It becomes two separate magnets with N and S.

Canadians and their government speak french in Quebec, what is the best reasoning for this?

Its primary goal was to ensure that Canadian citizens had access to federal services in the official language of their choice. It can ensure respect for English and French and ensure equality of status and equal rights and privileges as to their use in federal institutions;

Who is Louis Pasteur? Explain his contribution to science.

Louis Pasteur is best known for inventing the process that bears his name, pasteurization. Pasteurization kills microbes and prevents spoilage in beer, milk, and other goods. In his work with silkworms, Pasteur developed practices that are still used today for preventing disease in silkworm eggs.

conduct research on nutritional components of school lunch program; first resource?

MDE website

What U.S. economy employs the largest number of workers?

Manufacturing

Why are health textbooks important?

Medical textbooks are an important aid in the process of diagnosing and treating patients. Medical students use these books to acquire the skills necessary for this process, while medical teachers and experienced doctors use them for teaching these competences

capture the big picture when reading an article, best graphic organizer for helping students understand the big picture of information text

fact pyramid

What is the most abundant gas in the atmosphere?

Nitrogen

Uncle

Only capitalize when put in front of a name

Echinoderms

Organisms that have an endoskeleton and a water vascular system for movement, (ex. starfish, brittle stars, sea urchins, sand dollars, sea cucumbers, and crinoids) asexually, and sexually, they can be considered garbage

Base Map

field surveys to begin making maps

What do you do when beginning a play?

Read the play and discuss what you want to accomplish.

In music, what is rhythm?

Rhythm is music's pattern in time. Whatever other elements a given piece of music may have (e.g., patterns in pitch or timbre), rhythm is the one indispensable element of all music. Rhythm can exist without melody

How to find slope of a line

Rise/ run which means, the change in Y/ change in X

What is the most important thing for a visual text to do in order to convey meaning?

Sequence of Frames.

what skills do students develop from pre-k to elementary?

fine motor

David Blackwell

Statistician and mathematician, game theory, probability theory, info theory, and Bayesian and statistics

Scientific Method

Step 1- Question. The "thing" that you want to know. ... Step 2-Research. Conduct research. ... Step 3-Hypothesis. Educated guess or prediction of the outcome experiment. ... Step 4-Experiment. Test the hypothesis. Step 5-Observations. Data you collect during the experiment. Step6- Conclusion. Step 7- Communicate.

norm-referenced tests

Tests where a student's performance is compared with a norm group, or a representative sampling students similar to the student. A person's score on a norm-referenced test describes how the student did in relation to the norm group. Tests results are reported in such formats as standard scores or percentiles. (ex. SAT, IQ Tests, test that are graded on a curve, percentile rank)

How does the 1920s have an impact on today's society?

The 1920s represented an era of change and growth. The decade was one of learning and exploration. America had become a world power and was no longer considered just another former British colony. American culture, such as books, movies, and Broadway theater, was now being exported to the rest of the world.

Compare the Magna Carta and Mayflower Compact?

The English Magna Carta, written more than 400 years before the Mayflower Compact. The Mayflower Compact was a set of rules for self-governance established by the English settlers who traveled to the New World on the Mayflower. Magna Carta was issued in June 1215 and was the first document to put into writing the principle that the king and his government was not above the law. It sought to prevent the king from exploiting his power, and placed limits of royal authority by establishing law as a power in itself.

Find the value of square C.

The answer will be 100. All sides of each square are equal in length. 25* 4= 100

Phonemic Awareness

The awareness that spoken words are made up of speech sounds or phonemes.

Refraction

The bending of a wave as it passes at an angle from one medium to another (straw bending in water)

What happens to a country's import and export of countries currency falls?

The economics of supply and demand dictate that when demand is high, prices rise and the currency appreciates in value. In contrast, if a country imports more than it exports, there is relatively less demand for its currency, so prices should decline. In the case of currency, it depreciates or loses value.

What are the stages of early literacy development?

The five stages of literacy development include emergent literacy, alphabetic fluency, words and patterns, intermediate reading, and advanced reading.

Which of the following beliefs did the Ancient Greeks and the modern US share?

The idea that LAWS are critical to ensure societies remain stable

What is separation of powers?

The intent of separation of powers is to prevent the concentration of unchecked power and to provide for checks and balances, in which the powers of one branch of government is limited by the powers of another branch—to prevent abuses of power and avoid autocracy. The branches include: Legislative, Executive, and Judicial.

Know the parts of the water cycle

The test asks about the clouds- Condensation

alphabetic principle

The understanding that each phoneme of our language is represented by a graphic symbol.

Semantics

The meaning of words in a sentence

During a play the audience is having a hard time hearing the person standing downstage who is speaking to another person standing upstage, what should be done?

The persons face should always be towards the audience in a play, it will help the audience to here the actors clearer.

Why did Europeans migrate to the U.S. between the years 1880 & 1920?

The rapid industrialization and urbanization led to them seeking more economic opportunity. There were more than 20 million.

Phonics

The relationship between the letters of written language and the sounds of spoken language; A method of teaching beginners to read and pronounce words by teaching them the phonetic value of letters, letter groups, and syllables

Explain the rock cycle process.

The rock cycle is layers of rock compressed and compacted over a long period of time a. Magma cools and becomes insidious rocks b. Then weathering and erosion changes the rock into tiny pieces of rock, also known as sediment c. Compaction and cementation compress the smaller rocks together to create sedimentary rocks d. Heat and pressure create metamorphic rocks e. And the melting of those cause the rock to melt back to its original form, magma

Environmental print

The signs, labels and other print found in the home, school, or community

Explain the water cycle process.

The water cycle shows the continuous movement of water within the Earth and atmosphere. It is a complex system that includes many different processes. Liquid water evaporates into water vapor, condenses to form clouds, and precipitates back to earth in the form of rain and snow.

At age 3 what are toddlers able to do?

Their vocab is usually 200 or more words which can lead to string 3 to 4 sentences, able to talk about past experience and begin to use pretend play.

At age 2 what are toddlers able to do?

They can say about 20 to 50 or more words and can combine 2 words to make a simple sentence. ( ex. "baby crying," "Daddy big," "Mommy ball," "mommy throw ball.")

How did Ancient Greeks influence the U.s.?

They excluded women and immigrants in voting. They had no right or say on determining laws on foreign policy. The Greeks were first to create a democracy.

When water evaporates from the ocean surface, that water moves into the (A) asthenosphere (B) biosphere (C) lithosphere (D) atmosphere

This question asks you recognize the spheres of Earth. Although water vapor may be present in various spheres, when it evaporates from the ocean surface it directly moves into the atmosphere even if later that same water ends up in another sphere as water moves through the water cycle. The correct answer, therefore is (D).

The fact that Earth's axis is tilted helps explain which of the following? (A) The variations of the seasons in temperate regions (B) The existence of high and low tides (C) Earth's period of revolution of 365 days (D) Earth's period of rotation of 24 hours

This question asks you to apply your knowledge of Earth's tilt and Earth's motions. The tides are caused by the gravitational attraction of the Moon and the Sun and by Earth's rotation on its axis. Earth's tilt does not help explain its revolution or rotation. The variation of the seasons in temperate regions is due to Earth's tilt. For example, when the northern end of Earth's axis is pointed toward the Sun, it is approximately the beginning of summer in the northern hemisphere. The correct answer, therefore, is (A).

A rock picked up on a hillside was found to contain tiny pieces of seashells. Which of the following is the best explanation of how this rock was formed? (A) It was formed when sediments on the bottom of an ancient sea floor were deeply buried and subjected to great pressure for long periods of time. (B) It was formed on or near Earth's surface from magma or lava that flowed during a volcanic eruption. (C) It was formed when minerals deep inside Earth were subjected to great heat and pressure. (D) It was formed by seafloor spreading and erosion of the mid-ocean ridge deep in the ocean

This question asks you to apply your knowledge of rock formation and the processes of Earth's history to a single sample, a rock containing tiny pieces of seashells. The presence of seashells in a rock on a hillside indicates that the area where the rock formed was under water many years ago. During the time that the area was under water, sediments and shells became deeply buried by more sediments. The pressure compacted the sediments and the particles were cemented together, thus forming rock. The correct answer, therefore, is (A).

Which of the following statements best describes what is most likely to occur in a small hibernating animal when the surrounding air temperature drops slightly below freezing? (A) The animal will die. (B) The animal ceases all metabolic activity. (C) The animal will wake up, and then find food high in fat calories. (D) The animal increases its use of stored body fat for energy.

This question asks you to identify the effect of one condition (air temperature) on a hibernating animal. Hibernation is a state of winter dormancy associated with lowered body temperature and lowered metabolism. Animals hibernate mainly because there is not enough food in their environment in the winter. During hibernation, an animal's heartbeat and breathing rate slow down and its body temperature falls almost to that of its surroundings. Animals can survive on the food stored as fat in their bodies. The correct answer, therefore, is (D).

Which of the following is true about the location of most of Earth's volcanoes and earthquakes? (A) Randomly over Earth's surface (B) At the boundaries of Earth's tectonic plates (C) Toward the middle of the continents (D) Where tidal stresses caused by the Moon are greatest

This question asks you to identify the location of most of Earth's volcanoes and earthquakes. Most occur at the boundaries of Earth's tectonic plates, which are not randomly located over Earth's surface, or in the middle of continents. The gravitational effects of the Moon, which influence tides, do not have a big enough influence on the solid Earth to cause earthquakes or volcanism in certain areas. The correct answer, therefore, is (B).

When the Moon looks like a half disk as from Earth, the phase is called (A) full (B) quarter (C) old (D) new

This question asks you to identity one of the phases of the Moon. When the entire side of the Moon that faces Earth is illuminated the phase is called full. When none of the side of the Moon that faces Earth is illuminated the phase is called new. None of the phases are called "old". When half of the side of the Moon that faces Earth is illuminated the phase is called quarter (either first quarter or last quarter). The correct answer, therefore, is (B)

"Ancestors of modern giraffes stretched their necks to reach higher twigs. This eventually caused their offspring to be born with longer necks." Which of the following most accurately describes the statement above? (A) It is a good example of a species evolving within an isolated, rapidly changing environment. (B) It is a good example of how natural selection favors a trait that aids survival. (C) It is a misconception, because organisms cannot pass along acquired characteristics genetically. (D) It is a misconception, because improved diet rather than muscle movement usually leads to feeding-related traits such as longer necks.

This question asks you to recognize a common misconception about evolution. It was once thought that an organism's acclimation to the environment could be passed on to its offspring. This was called the "inheritance of acquired characteristics." This theory, however, is now known to be wrong. Only genetically determined traits can be passed on to offspring. The correct answer, therefore, is (C).

Which of the following is an example of chemical weathering? (A) Acid rain dissolving limestone and making rocks smaller (B) Running water breaking off pieces of rocks and making them smaller (C) Freezing and thawing water breaking rocks into smaller pieces (D) Rockslides that break rocks into smaller pieces

This question asks you to recognize an example of chemical weathering. Running water breaking off pieces of rock, freezing and thawing of water in cracks of rocks that causes rocks to break into smaller pieces, and the effect of rockslides are all examples of physical weathering. The chemical reaction of acid rain with limestone (mostly calcium carbonate), which dissolves the limestone, is an example of chemical weathering. The correct answer, therefore, is (A).

Temperature regulation of the human body by evaporation is a primary function of which of the following? (A) The intestines (B) The skin (C) The liver (D) The heart

This question asks you to recognize which structure in the human body has a primary role in regulating the body's temperature by evaporation. The correct answer is (B), since it is the skin that regulates temperature through sweat glands.

Under which of the following conditions would glaze (a thin coating of ice) form? (A) Ice crystals develop as they fall through the clouds. (B) As ice crystals fall, the temperature of the air remains cold and the crystals do not melt. (C) Rain falls on a surface that is colder than the freezing temperature of water. (D) Water droplets fall and freeze as they pass through colder air.

This question asks you to use your knowledge of different forms of precipitation. Options (A), (B), and (D) describe conditions that would result in snow or sleet. When snowflakes fall through a layer of relatively warm air and melt, then encounter a shallow layer of cold air, droplets of supercooled water can form. When these droplets hit a subfreezing surface, the water freezes and forms a thin layer of ice, glaze. The correct answer, therefore, is (C).

Of the following, which color star has the highest surface temperature? (A) Red (B) Blue (C) Orange (D) Yellow

This question asks you to use your knowledge of the surface temperature of stars. The stars with the lowest surface temperatures are red. The progression is red, orange, yellow, white-yellow, white, blue-white, and blue, with blue stars having the highest surface temperatures. The correct answer, therefore, is (B).

Which of the following statements best describes, in a simplified way, what occurs during the process of respiration in plants? (A) Plants use ATP, water, and carbon dioxide to produce sugar and oxygen. (B) Plants use sugar and oxygen to produce energy, water, and carbon dioxide. (C) Plants use water and oxygen to produce sugar and carbon dioxide. (D) Plants use water, sugar, and carbon dioxide to produce oxygen and ATP.

This question tests your knowledge of a basic process. During respiration in plants, a sugar is broken down to produce ATP, with water and carbon dioxide being by-products of the process. The correct answer, therefore, is (B).

Who is Gregory Mendel? Explain his contribution to science.

Through his careful breeding of garden peas, Gregor Mendel discovered the basic principles of heredity and laid the mathematical foundation of the science of genetics.

What is the best game in gym for hand-eye coordination?

Throwing bean bags into a target

What are running records used for?

To check fluency.

What is the common purpose of all governments?

To make sure all citizens follow rules so we all stay structured.

What can you do in a 2nd gd classroom when a student is confusing the letter sounds of c & g.

Try both pronunciations with phonics elements.

Digraphs

Two letters that stand for a single sound.

What are good aerobic exercises?

Walking and Running.

What do antibiotics do to your immune system?

Weaken the immune system for animals and humans.

what question can you ask to help you promote higher level thinking in a 4th grade classroom?

What elements make this novel historical fiction?

1/8 or 12.5%

What is the probability of tossing heads on a coin 3 times in a row?

Friction question about why a block slows down

When one object is sliding on another it starts to slow down due to friction. This means it loses energy. However, the energy doesn't disappear. It changes from moving energy (also call kinetic energy) to heat energy. This is why we rub our hands together when it's cold.

Climate and natural environment

Why do Canadians live close to the border?

Synonyms

Words that have similar meanings (Big:Large)

Homonym

Words that sound the same, but are spelled different

antonym

a word that means the opposite of another word Example: fat and skinny, young and old, happy and sad, etc.

High frequency words

are the 400 or so words that make up nearly 70% of reading selections

When selecting vocabulary from the informational text, which of the following approaches is most beneficial?

asking students to select words

What is the slope of y=-5x+7 a. 5 b.-5 c. 7 d. -7

b

second language acquisition

beginning: they may have 500 words to their receptive vocabulary, but they refrain from speaking, but will listen and may copy words down

Which of the following words is the best example to use when demonstrating structural analysis for vocabulary development?

bicyclist

The teacher asks students to detect any faulty reasoning in a poster advertisement. What level of reading comprehension is this

evaluative

Mr. Mathers is teaching his class about the Louisiana Purchase. What social science should Mr. Mathers connect to this history lesson in order to best explain the importance of the Louisiana Purchase to the United States?

geography

What are kids able to do at the ages of 3 to 5?

hand eye coordination, place direction, climb, balance, run, gallop, push, and pull

affect

have an effect on; make a difference to.

If a student is reading a sentence and changes the word "tree" with "fence" from the sentence, I was taking my dog on a walk and he saw and squirrel and chased it up a "fence".

he's replacing the word according to the context of the sentence

Political Deadlock

hindering a passage of law (ex: someone hiding evidence on criminal acts or destroying it).

a word that has the same spelling as another word but has a different sound and a different meaning.

homographs

dynamics

how loud or soft a piece of music is

What are kids able to do at the ages of 5 to 7?

improvement of handwriting tools, and scissors, able to focus more on school work

Potential to Kinetic Energy

is mechanical or stored energy from an object that comes from factors such as its position relative to others, internal stress, electric charge or its condition rather than motion. ex. Paper airplane

context

is the space in which a part of a text exists, including what comes before and after a piece of text

What is petrology?

is the study of rocks, including their composition, texture, structure, occurrence, mode of formation, and history.

What is geology?

it is the study of the planet Earth as it pertains to the composition, structure, and origin of its rocks.

KHDBDCM

kilo, hecto, deca, base(meter), deci, centi, milli

Territories

land that belongs to a nation but is not a state and is not represented in the national government. U.S. has 5.

computational fluency

more than quickly producing correct answers

spatial sequence

pattern of expository writing geometrical or geographical arrangement of ideas. Up, down, left, right, top, bottom, under, over etc. Might give reader small view of situation or person and as it develops the reader learns more and more

A fourth grade teacher wants to have students label each part of the electric circuits. Several ELLs are in the class. Which strategy is best for these students?

providing students with word banks containing key vocabulary from the lesson

an activity in which older students read aloud predictable or rhyming books to first grade students. This activity will most likely promote

reading fluency

Acid rain is caused by

release of sulfur by burning of fossil fuels, harmful effects on plants, aquatic animals, and infrastructure

fact and opinion statements. Which of the instructional strategies would be most beneficial to ensure and maintain learning?

repetition

Sound isolation

requires the student to determine the beginning, middle or ending sounds in a word.

sound matching

requires the student to match a word to a particular sound.

effect

result

Syntax

sentence structure, or grammar

Vaccines reduce the incidence of specific diseases by

stimulating the body's immune system response to the infection

vaccines reduce the incidence of specific diseases by

stimulating the body's immune system response to the infection

Inquiry-based learning

students are involved in their learning forming questions and investigating to build meaning so they can think independently, develop solutions and support a point of view

decodable text

text that is aligned with previously taught phonics skills and contains a limited number of high frequency words

phonological awareness

the ability to hear the distinctive sounds of letters

Phonemes

the basic units of sound in language

Breve

the curved line over a vowel letter in a word. This diacritical mark indicates that the vowel sound is short

• Phonological awareness

the knowledge that words have separate parts (syllables, onsets and rimes, phonemes)

What happens to the molecules in a metal pole when heated?

the molecules move fast when heated, the object expands, and takes up more space

Morphology

the study of a language's parts and how those parts interact

compund sentence

two or more independent clauses (I visited the beach, and I got a really bad sun burn) there are two parts that can stand on their own.

Which of the following strategies is the most beneficial for students during the publishing stage of the writing process?

typing work on a computer

Masks that are made in two different countries

used from natural resources

What is a member of the string family?

viola

What are the best activites to use with enlgish language learners?

visuals, prior knowledge, scaffolding, and cooperative learning activities.

When do you use a greater variety of sentence types when revising a paper.

when a sentence is choppy.

sight words

words recognized and pronounced immediately, not requiring the use of decoding or word analysis strategies.

Homographs

words that are spelled the same, but have different meanings

How to find the slope of two coordinate points

y2-y1/x2-x1

• Phoneme Manipulation

• The ability to add and/or substitute phonemes

Phonology

• the study of speech sounds

Phoneme

•The smallest unit of sound in speech.


Kaugnay na mga set ng pag-aaral

Unit 7- Reduction of Risk Potential

View Set

Nursing Pellico Ch 28 Renal 788-802

View Set

Intro to Networks Chapter 6 Review David Jackson

View Set

Simulation Lab 11.1: Module 11 Harden PC with Group Policy Editor

View Set

Chapter 2: What is good for society?

View Set